Clinical Cases in the Echocardiography Lab [1st ed.] 978-3-030-16617-5;978-3-030-16618-2

This book comprehensively covers unusual and rare pathological cases in echocardiography. Chapters cover cases from diag

437 54 10MB

English Pages XVI, 238 [210] Year 2019

Report DMCA / Copyright

DOWNLOAD FILE

Polecaj historie

Clinical Cases in the Echocardiography Lab [1st ed.]
 978-3-030-16617-5;978-3-030-16618-2

Table of contents :
Front Matter ....Pages i-xvi
Front Matter ....Pages 1-1
Quadricuspid Aortic Valve (Atooshe Rohani)....Pages 3-5
Unicuspid Aortic Valve (Atooshe Rohani)....Pages 7-9
Annulo-aortic Ectasia and Sinus of Valsalva Aneurysm (Atooshe Rohani)....Pages 11-13
Para-valvular Leak Secondary to Metallic Aortic Valve Endocarditis (Atooshe Rohani)....Pages 15-17
Severe Aortic Regurgitation (AI) (Atooshe Rohani)....Pages 19-22
Metallic Aortic Valve Obstruction (Atooshe Rohani)....Pages 23-25
Native AV Endocarditis (Atooshe Rohani)....Pages 27-28
Front Matter ....Pages 29-29
Parachute MV and Subvalvular Ring (Atooshe Rohani)....Pages 31-34
A Case of Flail MV, Review of Myxomatous Mitral Valve Disease (Atooshe Rohani)....Pages 35-41
MV Endocarditis (Atooshe Rohani)....Pages 43-44
Bioprosthetic MV obstruction (Atooshe Rohani)....Pages 45-47
Double Orifice MV (Atooshe Rohani)....Pages 49-51
Ischemic Mitral Regurgitation (MR) (Atooshe Rohani)....Pages 53-55
Metallic MV Paravalvular Leak (Atooshe Rohani)....Pages 57-59
Metallic MV Obstruction (Atooshe Rohani)....Pages 61-63
MV Stenosis (Atooshe Rohani)....Pages 65-67
Bioprosthetic MV Partial Dehiscence (Atooshe Rohani)....Pages 69-71
Flail Posterior MV Leaflet in the Context of Rheumatic Heart Disease (Atooshe Rohani)....Pages 73-75
Left Atrial Appendage (LAA) Clot (Atooshe Rohani)....Pages 77-80
Front Matter ....Pages 81-81
Ebstein Anomaly (Atooshe Rohani)....Pages 83-85
Papillary Fibroelastoma of TV (Atooshe Rohani)....Pages 87-88
TV Atresia (Atooshe Rohani)....Pages 89-91
TV Endocarditis (Atooshe Rohani)....Pages 93-94
Metallic Tricuspid Valve Obstruction (Atooshe Rohani)....Pages 95-97
Bioprosthetic TV Obstruction (Atooshe Rohani)....Pages 99-101
Front Matter ....Pages 103-103
Pulmonary Valve Stenosis (Atooshe Rohani)....Pages 105-107
Sub-valvular Pulmonary Stenosis (Atooshe Rohani)....Pages 109-111
Supra-valvular Pulmonary Stenosis (Atooshe Rohani)....Pages 113-116
Severe Pulmonary Insufficiency (PI) (Atooshe Rohani)....Pages 117-120
Pulmonary Valve Endocarditis (Atooshe Rohani)....Pages 121-123
Front Matter ....Pages 125-125
Non-compaction Cardiomyopathy (Atooshe Rohani)....Pages 127-129
Twenty Years After Hydatid Cyst Operation (Atooshe Rohani)....Pages 131-133
Amyloid Heart Disease (Atooshe Rohani)....Pages 135-138
Apical Hypertrophic Cardiomyopathy (aHCM) (Atooshe Rohani)....Pages 139-141
Hypertrophic Obstructive Cardiomyopathy (Atooshe Rohani)....Pages 143-147
Huge LV Apical Clot (Atooshe Rohani)....Pages 149-150
Source of Embolism to Brain (Atooshe Rohani)....Pages 151-152
Huge Calcified Mass, Inside Heart or Outside? (Atooshe Rohani)....Pages 153-155
Front Matter ....Pages 157-157
Coarctation of Aorta (Co-A) (Atooshe Rohani)....Pages 159-162
Patent Ductus Arteriosus Endocarditis (Atooshe Rohani)....Pages 163-166
Secundum Atrial Septal Defect (ASD) (Atooshe Rohani)....Pages 167-170
Atrioventricular Septal Defect (AVSD) (Atooshe Rohani)....Pages 171-173
L-Transposition of Great Arteries (L-TGA) (Atooshe Rohani)....Pages 175-179
Sinus Venosus ASD (Atooshe Rohani)....Pages 181-184
Left Anomalous Pulmonary Vein Connection (Atooshe Rohani)....Pages 185-187
Tetralogy of Fallot (Atooshe Rohani)....Pages 189-191
Single Ventricle with D-TGA and Pulmonary Stenosis (PS) (Atooshe Rohani)....Pages 193-196
Inlet Ventricular Septal Defect (VSD) (Atooshe Rohani)....Pages 197-200
Patent Foramen Ovale (PFO) (Atooshe Rohani)....Pages 201-203
Front Matter ....Pages 205-205
Arrhythmogenic Right Ventricular Cardiomyopathy (ARVC) (Atooshe Rohani)....Pages 207-209
Pulmonary Embolism (Atooshe Rohani)....Pages 211-214
Huge RV Myxoma (Atooshe Rohani)....Pages 215-217
Front Matter ....Pages 219-219
Aortic Dissection (Atooshe Rohani)....Pages 221-224
Pseudo Aneurysm of Dacron Graft of Aorta (Atooshe Rohani)....Pages 225-227
Front Matter ....Pages 229-229
Congenital Absence of Pericardium (Atooshe Rohani)....Pages 231-233
Back Matter ....Pages 235-238

Citation preview

Clinical Cases in Cardiology Series Editors: Ravi V. Shah · Siddique A. Abbasi · James L. Januzzi

Atooshe Rohani

Clinical Cases in the Echocardiography Lab

Clinical Cases in Cardiology Series editors: Ravi V. Shah Boston, MA, USA Siddique A. Abbasi Providence, RI, USA James L. Januzzi Boston, MA, USA

Clinical cases are a key component in modern medical education, assisting the trainee or recertifying clinician to work through unusual cases using best practice techniques. Cardiology is a key discipline in this regard and is a highly visual subject requiring the reader to describe often very subtle differences in the presentation of patients and define accurately the diagnostic and management criteria on which to base their clinical decision-making. This series of concise practical guides is designed to facilitate the clinical decisionmaking process by reviewing a number of cases and defining the various diagnostic and management decisions open to clinicians. Each title will be illustrated and diverse in scope, enabling the reader to obtain relevant clinical information regarding both standard and unusual cases in a rapid, easy to digest format. More information about this series at http://www.springer. com/series/14348

Atooshe Rohani

Clinical Cases in the Echocardiography Lab

Atooshe Rohani Northern Ontario School of Medicine Thunder Bay ON Canada

ISSN 2523-3009     ISSN 2523-3017 (electronic) Clinical Cases in Cardiology ISBN 978-3-030-16617-5    ISBN 978-3-030-16618-2 (eBook) https://doi.org/10.1007/978-3-030-16618-2 © Springer Nature Switzerland AG 2019 This work is subject to copyright. All rights are reserved by the Publisher, whether the whole or part of the material is concerned, specifically the rights of translation, reprinting, reuse of illustrations, recitation, broadcasting, reproduction on microfilms or in any other physical way, and transmission or information storage and retrieval, electronic adaptation, computer software, or by similar or dissimilar methodology now known or hereafter developed. The use of general descriptive names, registered names, trademarks, service marks, etc. in this publication does not imply, even in the absence of a specific statement, that such names are exempt from the relevant protective laws and regulations and therefore free for general use. The publisher, the authors, and the editors are safe to assume that the advice and information in this book are believed to be true and accurate at the date of publication. Neither the publisher nor the authors or the editors give a warranty, express or implied, with respect to the material contained herein or for any errors or omissions that may have been made. The publisher remains neutral with regard to jurisdictional claims in published maps and institutional affiliations. This Springer imprint is published by the registered company Springer Nature Switzerland AG The registered company address is: Gewerbestrasse 11, 6330 Cham, Switzerland

Acknowledgement

When I started my career as an echocardiologist, I was looking for a book like this to help me in determining a precise and evidence-based diagnosis in a challenging and busy echocardiography lab, so I start writing this book as a personal experience to guide echocardiologists, echocardiography fellows, and whoever wants a practical guidance for making a right diagnosis of challenging echocardiography cases. I very much appreciate all of my patients in Quaem Hospital, Mashhad, Iran, who gave me consent to present their echocardiography pictures and movies in this book. I would like to take the opportunity to acknowledge Carolyn Leonzio and Heather Seed’s assistance for the preparation of the proposal of this book. My wish for you is that you will find this book helpful and enjoy reading it as much as I enjoyed writing it.

v

Contents

Part I Aortic Valve (AV) 1

Quadricuspid Aortic Valve ������������������������������������������������  3 1.1 Echo Pearls . . . . . . . . . . . . . . . . . . . . . . . . . . . . . . . .   3 References . . . . . . . . . . . . . . . . . . . . . . . . . . . . . . . . . . . .   5

2

Unicuspid Aortic Valve��������������������������������������������������������  7 2.1 Echo Pearls���������������������������������������������������������������   7 References�����������������������������������������������������������������������   9

3

Annulo-aortic Ectasia and Sinus of Valsalva Aneurysm �������������������������������������������������������������� 11 3.1 Echo Pearls���������������������������������������������������������������  13 References�����������������������������������������������������������������������  13

4

Para-valvular Leak Secondary to Metallic Aortic Valve Endocarditis����������������������������������  15 4.1 Echo Pearls���������������������������������������������������������������  15 References�����������������������������������������������������������������������  17

5

Severe Aortic Regurgitation (AI)������������������������������������  19 5.1 Echo Pearls���������������������������������������������������������������  19 References�����������������������������������������������������������������������  22

6

Metallic Aortic Valve Obstruction ���������������������������������� 23 6.1 Echo Pearls���������������������������������������������������������������  23 References�����������������������������������������������������������������������  25

7

Native AV Endocarditis������������������������������������������������������ 27 7.1 Echo Pearls���������������������������������������������������������������  27 References�����������������������������������������������������������������������  28 vii

viii

Contents

Part II Mitral Valve (MV) 8

Parachute MV and Subvalvular Ring������������������������������ 31 8.1 Echo Pearls���������������������������������������������������������������  31 References�����������������������������������������������������������������������  34

9

A Case of Flail MV, Review of Myxomatous Mitral Valve Disease������������������������������������������������������������������������� 35 9.1 Mitral Valve Prolapse ���������������������������������������������  35 9.2  Echo Pearls���������������������������������������������������������������  35 References�����������������������������������������������������������������������  41

10

MV Endocarditis ������������������������������������������������������������������ 43 10.1 Echo Pearls�������������������������������������������������������������  43 Reference�������������������������������������������������������������������������  44

11 Bioprosthetic MV obstruction������������������������������������������ 45 11.1 Echo Pearls�������������������������������������������������������������  45 References�����������������������������������������������������������������������  47 12 Double Orifice MV�������������������������������������������������������������� 49 12.1 Echo Pearls�������������������������������������������������������������  49 References�����������������������������������������������������������������������  51 13 Ischemic Mitral Regurgitation (MR)������������������������������ 53 13.1 Echo Pearls�������������������������������������������������������������  54 References�����������������������������������������������������������������������  55 14 Metallic MV Paravalvular Leak���������������������������������������� 57 14.1 Echo Pearls�������������������������������������������������������������  57 References�����������������������������������������������������������������������  59 15 Metallic MV Obstruction���������������������������������������������������� 61 15.1 Echo Pearls�������������������������������������������������������������  61 References�����������������������������������������������������������������������  62 16

MV Stenosis���������������������������������������������������������������������������� 65 16.1 Physical Examination �������������������������������������������  65 16.2 Echo Pearls�������������������������������������������������������������  66 References�����������������������������������������������������������������������  67

17 Bioprosthetic MV Partial Dehiscence���������������������������� 69 17.1 Echo Pearls �������������������������������������������������������������  71 References�����������������������������������������������������������������������  71

Contents

ix

18 Flail Posterior MV Leaflet in the Context of Rheumatic Heart Disease ���������������������������� 73 18.1 Echo Pearls�������������������������������������������������������������������� 73 References ������������������������������������������������������������������������������  74 19 Left Atrial Appendage (LAA) Clot�������������������������������� 77 19.1 Echo Pearls�������������������������������������������������������������  77 References�����������������������������������������������������������������������  80 Part III Tricuspid Valve (TV) 20

Ebstein Anomaly������������������������������������������������������������������ 83 20.1 Echo Pearls�������������������������������������������������������������  85 References�����������������������������������������������������������������������  85

21 Papillary Fibroelastoma of TV������������������������������������������ 87 21.1 Echo Pearls�������������������������������������������������������������  87 References�����������������������������������������������������������������������  88 22

TV Atresia ������������������������������������������������������������������������������ 89 22.1 Echo Pearls�������������������������������������������������������������  89 References�����������������������������������������������������������������������  91

23

TV Endocarditis�������������������������������������������������������������������� 93 23.1 Echo Pearls�������������������������������������������������������������  93 References�����������������������������������������������������������������������  94

24 Metallic Tricuspid Valve Obstruction������������������������������ 95 24.1 Echo Pearls�������������������������������������������������������������  95 References�����������������������������������������������������������������������  96 25

Bioprosthetic TV Obstruction������������������������������������������ 99

Part IV Pulmonic Valve (PV) 26

Pulmonary Valve Stenosis��������������������������������������������������105 26.1 Echo Pearls������������������������������������������������������������� 105 References����������������������������������������������������������������������� 107

27 Sub-valvular Pulmonary Stenosis��������������������������������� 109 27.1 Echo Pearls for Subvalvular PS ��������������������������� 111 References����������������������������������������������������������������������� 111

x

Contents

28 Supra-valvular Pulmonary Stenosis����������������������������� 113 28.1 Echo Pearls������������������������������������������������������������� 116 References����������������������������������������������������������������������� 116 29 Severe Pulmonary Insufficiency (PI)��������������������������� 117 29.1 Echo Pearls������������������������������������������������������������� 119 29.1.1 Severe Pi Echocardiographic Signs��������� 119 References����������������������������������������������������������������������� 120 30

Pulmonary Valve Endocarditis������������������������������������� 121 30.1 Echo Pearls������������������������������������������������������������� 121 References����������������������������������������������������������������������� 123

Part V Left Ventricle (LV) 31

Non-compaction Cardiomyopathy������������������������������� 127 31.1 Echo Pearls ����������������������������������������������������������� 127 References����������������������������������������������������������������������� 129

32 Twenty Years After Hydatid Cyst Operation������������� 131 32.1 Echo Pearls ����������������������������������������������������������� 132 References����������������������������������������������������������������������� 133 33 Amyloid Heart Disease������������������������������������������������� 135 33.1 Echo Pearls������������������������������������������������������������� 137 References����������������������������������������������������������������������� 138 34 Apical Hypertrophic Cardiomyopathy (aHCM) ������� 139 34.1 Echo Pearls������������������������������������������������������������� 140 References����������������������������������������������������������������������� 141 35 Hypertrophic Obstructive Cardiomyopathy��������������� 143 35.1 Echo Pearls������������������������������������������������������������� 145 References����������������������������������������������������������������������� 147 36 Huge LV Apical Clot����������������������������������������������������� 149 36.1 Echo Pearls������������������������������������������������������������� 149 References����������������������������������������������������������������������� 150 37 Source of Embolism to Brain��������������������������������������� 151 37.1 Echo Pearls������������������������������������������������������������� 151 References����������������������������������������������������������������������� 152

Contents

xi

38 Huge Calcified Mass, Inside Heart or Outside?��������� 153 38.1 Echo Pearls������������������������������������������������������������� 155 References����������������������������������������������������������������������� 155 Part VI Congenital Heart Disease 39 Coarctation of Aorta (Co-A) ��������������������������������������� 159 39.1 Echo Pearls������������������������������������������������������������� 161 References����������������������������������������������������������������������� 162 40 Patent Ductus Arteriosus Endocarditis����������������������� 163 40.1 Echo Pearls������������������������������������������������������������� 163 References����������������������������������������������������������������������� 166 41 Secundum Atrial Septal Defect (ASD)����������������������� 167 41.1 Echo Pearls������������������������������������������������������������� 167 References����������������������������������������������������������������������� 170 42 Atrioventricular Septal Defect (AVSD) ��������������������� 171 42.1 Echo Pearls������������������������������������������������������������� 171 References����������������������������������������������������������������������� 173 43 L-Transposition of Great Arteries (L-TGA) ������������� 175 43.1 Echo Pearls for Congenital Heart Disease; Step by Step Echocardiographic Approach������� 177 43.2 Next Steps��������������������������������������������������������������� 178 43.3 LTGA ��������������������������������������������������������������������� 179 References����������������������������������������������������������������������� 179 44 Sinus Venosus ASD ������������������������������������������������������� 181 44.1 Echo Pearls������������������������������������������������������������� 181 References����������������������������������������������������������������������� 184 45 Left Anomalous Pulmonary Vein Connection����������� 185 45.1 Echo Pearls������������������������������������������������������������� 185 References����������������������������������������������������������������������� 187 46 Tetralogy of Fallot ��������������������������������������������������������� 189 46.1 Echo Pearls������������������������������������������������������������� 189 References����������������������������������������������������������������������� 191 47 Single Ventricle with D-TGA and Pulmonary Stenosis (PS)����������������������������������������������� 193 47.1 Echo Pearl��������������������������������������������������������������� 195 References����������������������������������������������������������������������� 196

xii

Contents

48 Inlet Ventricular Septal Defect (VSD) ����������������������� 197 48.1 Echo Pearls������������������������������������������������������������� 197 References�����������������������������������������������������������������������200 49 Patent Foramen Ovale (PFO)���������������������������������������201 49.1 Echo Pearls�������������������������������������������������������������201 References����������������������������������������������������������������������� 203 Part VII Right Ventricle (RV) 50

Arrhythmogenic Right Ventricular Cardiomyopathy (ARVC)��������������������������������������������� 207 50.1 Echo Pearls������������������������������������������������������������� 207 50.2 Major Echocardiographic Criteria for ARVC (Arrythmogenic RV Cardiomyopathy)�������������� 208 50.3 Minor Echocardiographic Criteria for ARVC��� 208 50.4 Echo Pearl ������������������������������������������������������������� 209 References����������������������������������������������������������������������� 209

51

Pulmonary Embolism ��������������������������������������������������� 211 51.1 Echo Pearls������������������������������������������������������������� 212 References����������������������������������������������������������������������� 214

52 Huge RV Myxoma��������������������������������������������������������� 215 52.1 Echo Pearls������������������������������������������������������������� 216 References����������������������������������������������������������������������� 217 Part VIII AORTA 53

Aortic Dissection ����������������������������������������������������������� 221 53.1 Echo Pearls������������������������������������������������������������� 223 References����������������������������������������������������������������������� 224

54 Pseudo Aneurysm of Dacron Graft of Aorta������������� 225 54.1 Echo Pearls������������������������������������������������������������� 227 References����������������������������������������������������������������������� 227 Part IX Pericardium 55 Congenital Absence of Pericardium���������������������������� 231 55.1 Echo Pearls������������������������������������������������������������� 232 References����������������������������������������������������������������������� 233 Index����������������������������������������������������������������������������������������� 235

Introduction

Clinical cases in the echocardiography lab addresses various common, uncommon, and complex echocardiography cases. Echocardiography fellows preparing for the Board of Echocardiography examination and cardiologists are bound to gain from this book. The book follows a wide-ranging, convenient, and illustrated course. I choose echocardiography cases that may not be found easily in any other resources. All cases described in this book originate from my daily personal experience of working as an echocardiologist in a busy and challenging echocardiography lab in an academic hospital. Echocardiography cases in echocardiography lab have nine parts; every part has chapters; in each chapter, you will find a case presentation with a brief history, physical examination, echocardiographic details, and echocardiography movies, pictures, and tables. At the end of each chapter, you will find echocardiographic pearls which are evidence-based echocardiographic features of the case, based on the literature review, my personal experience, and the American Society of Echocardiography guidelines. For transesophageal echocardiogram, you will find practical points about obtaining specific views which are from my personal experience in addition to common standard views. There are also interesting three-dimensional echocardiography pictures and movies which are very helpful to establish the correct diagnosis, especially for prosthetic heart valve complications, intracardiac masses, and congenital heart disease. xiii

xiv

Introduction

As an adult cardiologist, dealing with congenital heart disease is always challenging. This book has specific chapter about all common topics as well as some complex congenital cases; I tried to explain a step-by-step guide for echocardiographic interpretation of these cases. I hope, by writing this book, I could transfer my echocardiography expertise to all readers. I have inspired and written this book with love and with the wish of helping my colleagues across the world to make echocardiographic interpretation easy, enjoyable, and evidence based. Love is an echo which reflects only your essence, if you have the courage to look in its face. Rumi

Abbreviations

3D Three-dimensional AHCM Apical hypertrophic cardiomyopathy AR Aortic regurgitation ASD Atrial septal defect AVSD Atrioventricular septal defect bpm Beats per minute BSA Body surface area CW Continuous-wave Doppler D-TGA D-transposition of great arteries HOCM Hypertrophic obstructive cardiomyopathy JVP Jugular venous pulse LA Left atrium LAA Left atrial appendage L-TGA L-transposition of great arteries LV Left ventricle LVOT Left ventricular outflow MR Mitral regurgitation MV Mitral valve NYHA New York Heart Association class PLAX Parasternal long axis PPM Patient prosthesis mismatch PSAX Parasternal short axis PV Pulmonary valve PVL Paravalvular leakage PW Pulse-wave Doppler RA Right atrium RHD Rheumatic heart disease xv

xvi

Abbreviations

RV RVH RVOT TEE TR TTE TV TVI

Right ventricle Right ventricular hypertrophy Right ventricular outflow Transesophageal echocardiography Tricuspid regurgitation Transthoracic echocardiography Tricuspid valve Time velocity integral

Part I

Aortic Valve (AV)

Chapter 1 Quadricuspid Aortic Valve

A 38  years old man presented with exertional shortness of breath. On examination, blood pressure was 136/52  mmHg. Heart had a diastolic murmur grade 2/6  in the second right inter-costal space. TTE showed normal left ventricular chamber size and systolic function. There was moderate aortic regurgitation (AR) with valve anatomy suspicious of Quadricuspid Aortic Valve (QAV). TEE performed for evaluation of aortic regurgitation mechanism. The aortic valve was found to be quadricuspid; type A (Fig. 1.1, Movie 1.1).

1.1  Echo Pearls 1. Hurwitz and Roberts [1] classified QAV into seven groups: • Type A – four equal cusps. • Type B – three equal cusps and one smaller cusp. • Type C – two equal larger cusps and two equal smaller cusps.

Electronic Supplementary Material  The online version of this chapter (https://doi.org/10.1007/978-3-030-16618-2_1) contains supplementary material, which is available to authorized users.

© Springer Nature Switzerland AG 2019 A. Rohani, Clinical Cases in the Echocardiography Lab, Clinical Cases in Cardiology, https://doi.org/10.1007/978-3-030-16618-2_1

3

4

Chapter 1.  Quadricuspid Aortic Valve

Figure 1.1  Type A quadricuspid AV (four equal leaflets) 45 degree TEE mid esophageal

• Type D  – one large, two intermediate, and one small cusp. • Type E – three equal cusps and one larger cusp. • Type F  – two equal larger cusps and two unequal smaller cusps. • Type G – four unequal cusps. 2. Echocardiographic signs of severe aortic regurgitation are as follows [2]: • LV is usually dilated, except for acute severe AI. • Aortic leaflets are abnormal/flail or there is wide coaptation defect. • Holodiastolic reversal in abdominal aorta and descending thoracic aorta (Fig. 1.2) is a sign of significant aortic regurgitation. • In case of severe AI, Time Velocity Integral (TVI) of backflow signal in descending thoracic aorta is equal to TVI of LVOT.

References

5

Figure 1.2  Red arrow shows back flow (diastolic flow reversal) in schematic pulse wave doppler of descending thoracic aorta

References 1. Hurwitz LE, Roberts WC.  Quadricuspid semilunar valve. Am J Cardiol. 1973;31(5):623–6. 2. Garg P, Kamaruddin H, Orme R, Watt V. Type F congenital quadricuspid aortic valve: A very rare case diagnosed by 3-dimenional transoesophageal echocardiography. Open Cardiovasc Med J. 2014;8:23–5.

Chapter 2 Unicuspid Aortic Valve

Patient is a 20 years old young man who presented with dyspnea on exertion and palpitation from 2 months ago. He had history of successful stent deployment for coarctation of aorta during middle childhood. Physical examination revealed normal vital sign and systolic murmur grade 3/6 in the aortic area. Echocardiography revealed unicuspid aortic valve (UAV) (Fig.  2.1, Movie 2.1) with an eccentric orifice, and aortic valve area of 0.9  cm2. There is no echocardiographic evidence of re-coarctation. (Fig. 2.2) He underwent successful aortic valve replacement.

2.1  Echo Pearls [1, 2] 1. There are two subtypes of UAV: Slit shaped and pinhole. Pin hole UAV has early presentation during infancy. 2. Always pay attention to the morphology of aortic valve, high gradient in LVOT (left ventricular outflow) is not equal to aortic valve stenosis, it could be the result of sub-­ valvular web or hypertrophic obstructive cardiomyopathy.

Electronic Supplementary Material The online version of this chapter (https://doi.org/10.1007/978-3-030-16618-2_2) contains supplementary material, which is available to authorized users.

© Springer Nature Switzerland AG 2019 A. Rohani, Clinical Cases in the Echocardiography Lab, Clinical Cases in Cardiology, https://doi.org/10.1007/978-3-030-16618-2_2

7

8

Chapter 2.  Unicuspid Aortic Valve

Figure 2.1 TEE short axis view of unicuspid aortic valve, mid esophageal 55 degree

Figure 2.2  TEE midesophageal shows stent in descending thoracic aorta

3. Aortic valve peak velocity more than 4 m/s, mean pressure gradient more than 40 mmHg and Doppler velocity index (LVOT VTI divided by AV VTI) less than 0.25 in the context of abnormal morphology of aortic valve (Calcific aortic

References

9

valve stenosis (CAVS), bicuspid, unicuspid vave) are echocardiographic criteria for severe aortic valve stenosis.

References 1. Nishimura RA, Otto CM, Bonow RO, Carabello BA, Erwin JP, Fleisher L, et  al. 2017 AHA/ACC focused update of the 2014 AHA/ACC guideline for the management of patients with valvular heart disease: a report of the American College of Cardiology/ American Heart Association task force on clinical practice guidelines. Circulation. 2017;135:e1159–95. 2. Mookadam F, Thota VR, Lopez AM, Emani UR, Tajik AJ. Unicuspid aortic valve in children: a systematic review spanning four decades. J Heart Valve Dis. 2010;19:678–83.

Chapter 3 Annulo-aortic Ectasia and Sinus of Valsalva Aneurysm A 62-year-old man presented with vague chest pain at rest associated with progressive shortness of breath over the last year. Patient does not have clinical features or family history of Marfan syndrome. Physical examination revealed regular heart rate of 80 beats per minute, and blood pressure of 150/47  mm Hg. A diastolic murmur grade 2/6 heard in the right second intercostals space. Electrocardiography (ECG) showed ST-segment depression in V2–V6. Chest radiography showed cardiomegaly. Trans-thoracic echocardiography revealed severely increased LV size, left atrial enlargement, severe aortic regurgitation (AI) and dilation of the sinus of valsalva and aortic annulus. Transesophageal echocardiography (TEE) demonstrated aneurysmal dilatation of the non-coronary cusp. (Fig. 3.1, 3.2, Movie 3.1).

Electronic Supplementary Material The online version of this chapter (https://doi.org/10.1007/978-3-030-16618-2_3) contains supplementary material, which is available to authorized users.

© Springer Nature Switzerland AG 2019 A. Rohani, Clinical Cases in the Echocardiography Lab, Clinical Cases in Cardiology, https://doi.org/10.1007/978-3-030-16618-2_3

11

12

Chapter 3.  Annulo-aortic Ectasia and Sinus of Valsalva...

Figure 3.1 TEE mid esophageal shows dilation of non-coronary sinus of valsalva and proximal ascending aorta

Figure 3.2  Holo diastolic flow reversal in descending thoracic aorta

References

13

3.1  Echo Pearls [1, 2] 1. Sinus of Valsalva aneurysm is defined as asymmetric dilation one of sinuses. 2. The most common location is the right sinus, then non-­ coronary sinus. 3. In case of aortic aneurysm, there is effacement of sinotubular junction. 4. Search for associated anomalies like as ventricular septal defect (VSD). 5. David’s operation for annuloaortic ectasia and aortic regurgitation consist of excising the aneurysmal portion of the ascending aorta and sinuses of Valsalva and leaving the aortic valve leaflets in place; parts of aortic wall remain connected to the left ventricular outflow tract inside the Dacron tube.

References 1. Beyersdorf F, Rylski B. Current state of the reimplantation technique (DAVID operation): surgical details and results. HSR Proc Intensive Care Cardiovasc Anesth. 2012;4(2):73–6. 2. Ring WS. Congenital Heart Surgery Nomenclature and Database Project: aortic aneurysm, sinus of Valsalva aneurysm, and aortic dissection. Ann Thorac Surg. 2000;69(suppl 1):S147–63.

Chapter 4 Para-valvular Leak Secondary to Metallic Aortic Valve Endocarditis Patient was a 40-year-old man with a history of metalic aortic valve replacement who presented with fever, chills and shortness of breath. The patient had a mechanical aortic valve for rheumatic disease 10 months before this presentation. On examination blood pressure was 118/65 mmHg, heart rate of 103 beats/min, respiratory rate of 22 breaths/min, and oxygen saturation of 100% on room air. He had clear chest with no adventitious sound, and a high-pitched decrescendo murmur heard at the left sternal border in early diastole. Laboratory analysis showed no evidence of hemolysis. Transesophageal echocardiogram (TEE) revealed a mobile mass in the aortic side of valve (Fig. 4.1) and moderate paravalvular leakage.

4.1  Echo Pearls [1–3] • The severity of aortic para-valvular regurgitation is defined by defined aortic regurgitation criteria such as jet width (vena contracta), jet density, jet pressure half time (PHT) and diastolic flow reversal in descending aorta. We could also trace the area of regurgitation in short axis view of great vessels, and divide it to the whole aortic valve area [1].

© Springer Nature Switzerland AG 2019 A. Rohani, Clinical Cases in the Echocardiography Lab, Clinical Cases in Cardiology, https://doi.org/10.1007/978-3-030-16618-2_4

15

16

Chapter 4.  Para-valvular Leak Secondary to Metallic…

Figure 4.1  TEE 96 degree shows mobile mass in aortic side of valve

Posterior 12

Interatrial septum Medial

L 9

3 lateral

N R 6 Anterior

Figure 4.2 Leak location from a surgeon’s view. (schematic TEE short axis 60 degree, mid esophageal)

• Leak location is reported in a clock-wise manner from a surgical view (TEE short axis 60 degree, mid esophageal). (Fig. 4.2). 1. Five o’clock is the commissure between the left and right coronary sinus.

References

17

2. 8 o’clock is the commissure between the right and non-­ coronary sinus. 3. 11 o’clock is the commissure between non-coronary and left coronary sinus. • Aortic PVLs are more commonly located between the right and non-coronary cusps at 8 o’clock.

References 1. Lancellotti P, Pibarot P, Chambers J, Edvardsen T, Delgado V, Dulgheru R, et  al. Recommendations for the imaging assessment of prosthetic heart valves: a report from the European Association of Cardiovascular Imaging endorsed by the Chinese Society of Echocardiography, the Inter-American Society of Echocardiography, and the Brazilian Department of Cardiovascular Imaging. Eur Heart J Cardiovasc Imaging. 2016;17(6):589–90. 2. Shanmugam G, MacArthur K, Pollock J. Mechanical aortic valve replacement: long-term outcomes in children. J Heart Valve Dis. 2005;14:166–71. 3. Schaff HV, Carrel TP, Jamieson WR, et al. Paravalvular leak and other events in silzone-coated mechanical heart valves (a report from AVERT). Ann Thorac Surg. 2002;73(3):785–92.

Chapter 5 Severe Aortic Regurgitation (AI)

A 50-year-old woman, presented with progressively worsening shortness of breath. She was known for rheumatoid arthritis. On examination pulse was 72 beats per minute and blood pressure was 115/52 mmHg. She had bounding carotid pulses and III/VI diastolic decrescendo murmur heard at the right second inter-costal space. She had bibasilar rales on lung auscultation. TTE and TEE showed thickened cusp, wide mal-­coaptation of aortic valve and severe AR) (Figs. 5.1, 5.2, and 5.3, Movie 5.1).

5.1  Echo Pearls [1, 2] Echo findings consistent with severe aortic regurgitation (AR) include: 1. Incomplete coaptation of the aortic valve with normal cusps secondary to aortic dilatation. (Fig. 5.1), or cusp perforation, cusp restriction and cusp prolapse. 2. Area of the color jet width of AR is 66% or more of the left ventricular outflow tract area. 3. PHT of the aortic regurgitant jet is less than 200 ms (Fig. 5.2) Electronic Supplementary Material The online version of this chapter (https://doi.org/10.1007/978-3-030-16618-2_5) contains supplementary material, which is available to authorized users.

© Springer Nature Switzerland AG 2019 A. Rohani, Clinical Cases in the Echocardiography Lab, Clinical Cases in Cardiology, https://doi.org/10.1007/978-3-030-16618-2_5

19

20

Chapter 5.  Severe Aortic Regurgitation (AI)

Figure 5.1  Malcoaptation of the aortic valve. (Red arrow)TEE 36 degree mid esophageal

Figure 5.2  PHT of the aortic regurgitant jet 85% for severe MR but is an insensitive echocardiographic sign (Fig. 9.6). • Regurgitant jet direction. • Left atrium size. • PISA (proximal isovelocity acceleration) distance(r) from colored area on the ventricular side of the mitral valve in systole to the center of the mitral regurgitant orifice (Fig.  9.4) EROA (effective regurgitant orifice area) = 2 × 3.14 × r2 × Nyquist Limit/Vmax • RV(REGURGITANT VOLUME) = EROA × VTI MR. 4. In case of MV prolapse (MVP), if regurgitation time is limited to the late systole, it is not severe; holosystolic jet is a sign of severe MR (Table 9.2).

References

41

References 1. Zoghbi W, Adams D, Bonow R, Enriquez-Sarano M, Foster E, Paul A, et  al. Recommendations for noninvasive evaluation of native valvular regurgitation a report from the American Society of Echocardiography developed in collaboration with the Society for Cardiovascular Magnetic Resonance. J Am Soc Echocardiogr. 2017;30:304–71. 2. Recusani F, Bargiggia GS, Yoganathan AP, Raisaro A, Valdes-­ Cruz LM, Sung HW, Bertucci C, Gallati M, Moises VA, Simpson IA.  A new method for quantification of regurgitant flow rate using color Doppler flow imaging of the flow convergence region proximal to a discrete orifice. An in  vitro study. Circulation. 1991;83(2):594–604.

Chapter 10 MV Endocarditis

A 54  years old man presents with fever, (39.9  °C) splenomegaly and apical systolic murmur. Investigation revealed leukocytosis and C-reactive protein (CRP) 187 mg/L. Three sets of blood cultures showed methicillin-sensitive staphylococcus aureus. TTE and TEE revealed multiple small mobile vegetations on the atrial side of posterior mitral valve leaflet with severe MR (Figs. 10.1 and 10.2; Movie 10.1).

10.1  Echo Pearls [1] 1. If initial exam is negative in the context of high clinical suspicion for endocarditis, consider repeating echo in 7-10 day. 2. Echo report for MV endocarditis should include LV size, presence or absence of pericarial effusion, pulmonary hypertension, perivalvular involvement, any thickening of valvular ring, vegetation size and mobility of vegetation.

Electronic Supplementary Material The online version of this chapter (https://doi.org/10.1007/978-3-030-16618-2_10) contains supplementary material, which is available to authorized users.

© Springer Nature Switzerland AG 2019 A. Rohani, Clinical Cases in the Echocardiography Lab, Clinical Cases in Cardiology, https://doi.org/10.1007/978-3-030-16618-2_10

43

44

Chapter 10.  MV Endocarditis

Figure 10.1 Multiple mobile vegetations on atrial side of mitral valve (MV) TEE 43 degree mid-esophgeal

Figure 10.2  3D TEE shows multiple vegetations (enface view)

Reference 1. Firschke C, Schömig A. Mitral-valve endocarditis. N Engl J Med. 2001;345(10):739.

Chapter 11 Bioprosthetic MV obstruction

Patient is a 33 years old lady who presents with shortness of breath, worsened gradually over the last year; New  York Heart Association [NYHA] functional class III. She is known for rheumatic heart disease and has a bioprosthetic mitral valve implanted 7 years ago. She denies chest pain. On examination, she has elevated JVP, irregular heart rhythm with the heart rate of 120 bpm, basilar crackles and diastolic rumble on heart auscultation. 2D and 3D TTE and TEE showed severe bioprosthetic valve stenosis, severe para valvular leakage and moderate pulmonary hypertension (Fig. 11.1 and Movie 11.1).

11.1  Echo Pearls [1–3] 1. Thrombus or pannus formation could lead to bioprosthetic obstruction. 2. Peak velocity of 2.5  m/s, mean gradient more than 10 mmHg, ratio of the time velocity integral of the mitral valve prosthesis divided by time velocity integral of left ventricular outflow tract >2.5 and pressure half time(PHT) Electronic Supplementary Material The online version of this chapter (https://doi.org/10.1007/978-3-030-16618-2_11) contains supplementary material, which is available to authorized users.

© Springer Nature Switzerland AG 2019 A. Rohani, Clinical Cases in the Echocardiography Lab, Clinical Cases in Cardiology, https://doi.org/10.1007/978-3-030-16618-2_11

45

46

Chapter 11.  Bioprosthetic MV obstruction

Figure 11.1  3D TEE echo shows degenerative leaflets Table 11.1  Echo characteristic of prosthetic obstruction, mismatch and pathologic regurgitation Clues for Patient – High mean Bioprosthetic prosthetic Pathologic gradient MV obstruction mismatch regurgitation PHT >130 1.9 m/s. • Mean MV gradient >5 mmHg. • Velocity-time integral (VTI) across the valve divided by left ventricular outflow tract (LVOT) VTI ratio >2.5. • Maximum velocity of tricuspid regurgitation >3 m/s.

Electronic Supplementary Material  The online version of this chapter (https://doi.org/10.1007/978-3-030-16618-2_14) contains supplementary material, which is available to authorized users.

© Springer Nature Switzerland AG 2019 A. Rohani, Clinical Cases in the Echocardiography Lab, Clinical Cases in Cardiology, https://doi.org/10.1007/978-3-030-16618-2_14

57

58

Chapter 14.  Metallic MV Paravalvular Leak

Figure 14.1 TEE 50 degree, 2 chamber view shows paravalvular leakage

Figure 14.2  Metallic valve dehiscence clearly seen by 3d TEE 112 degree midesophageal view (red arrows)

References

59

2. Echocardiographic report should provide following information: • Presence of pulmonary vein flow reversal. • Pulmonary artery systolic pressure. • Measurement of the ratio of total sewing ring circumference to the length of sutures dehiscence (20% - severe and >40% - instability of the prosthetic valve). 3. By convention, surgical or “en face” view of mitral valve is the view from the left atrial perspective. From this look a “clock face” referred to the aortic valve anterior to the mitral valve at 12 o’clock position and the left atrial appendage at 9 o’clock location (Fig. 14.2).

References 1. Perk G, Kronzon I. Interventional echocardiography in structural heart disease. Curr Cardiol Rep. 2013;15(3):338. 2. Kliger C, Eiros R, Isasti G, Einhorn B, Jelnin V, Cohen H, et al. Review of surgical prosthetic paravalvular leaks: diagnosis and catheter-based closure. Eur Heart J. 2013;34(9):638–49. 3. Kronzon I, Sugeng L, Perk G, Hirsh D, Weinert L, Garcia Fernandez MA, Lang RM. Real-time-3-dimensional transesophageal echocardiography in the evaluation of post-operative mitral annuloplasty ring and prosthetic valve dehiscence. J Am Coll Cardiol. 2009;53(17):1543–7.

Chapter 15 Metallic MV Obstruction

Patient was a 33  year old lady presented with worsening shortness of breath over the last three months; she had metallic mitral valve for over 10  years. Her prothrombin timeinternational normalized ration (PT-INR) values had ranged from 1.8 to 2.5, and her warfarin treatment time in the therapeutic range was around 20%. Transthoracic echocardiography showed mean pressure gradient of 18  mmHg across mitral valve and one fixed leaflet (Fig. 15.1 and Movie 15.1).

15.1  Echo Pearls [1–3] 1. It is very important to look at the leaflet motion of prosthetic valve in multiple views and angles on TEE examination. 2. Table 15.1 summarizes echo characteristic of thrombus and pannus formation on prosthetic valve.

Electronic Supplementary Material The online version of this chapter (https://doi.org/10.1007/978-3-030-16618-2_15) contains supplementary material, which is available to authorized users.

© Springer Nature Switzerland AG 2019 A. Rohani, Clinical Cases in the Echocardiography Lab, Clinical Cases in Cardiology, https://doi.org/10.1007/978-3-030-16618-2_15

61

62

Chapter 15.  Metallic MV Obstruction

Figure 15.1  Red arrow shows one fix leaflet, blue arrow shows normal leaflet Table 15.1 Thrombus VS pannus echocardiographic diagnostic criteria Thrombus Pannus Size Large, extends into No extension, the left atrium limited to the valve area Intensity of the mass

Soft echodensity, similar to myocardium

Dense

Anticoagulation

Inadequate

Adequate

Presence of mobile portions

Fixed lesion with no mobile component

References 1. Zoghbi WA, Chambers JB, Dumesnil JG, Foster E, Gottdiener JS, Grayburn PA, et al. Recommendations for evaluation of prosthetic valves with echocardiography and doppler ultrasound: a report from the American Society of Echocardiography’s Guidelines and Standards Committee and the task force on prosthetic valves,

References

63

developed in conjunction with the American College of Cardiology Cardiovascular Imaging Committee, Cardiac Imaging Committee of the American Heart Association, the European Association of Echocardiography, a registered branch of the European Society of Cardiology, the Japanese Society of Echocardiography and the Canadian Society of Echocardiography, endorsed by the American College of Cardiology Foundation, American Heart Association, European Association of Echocardiography, a registered branch of the European Society of Cardiology, the Japanese Society of Echocardiography, and Canadian Society of Echocardiography. J Am Soc Echocardiogr. 2009;22(9):975–1014. 2. Roudaut R, Serri K, Lafitte S.  Thrombosis of prosthetic heart valves: diagnosis and therapeutic considerations. Heart. 2007;93(1):137–42. 3. Toker ME, Eren E, Balkanay M, Kirali K, Yanartaş M, Calişkan A, Güler M, Yakut C. Multivariate analysis for operative ­mortality in obstructive prosthetic valve dysfunction due to pannus and thrombus formation. Int Heart J. 2006;47(2):237–45.

Chapter 16 MV Stenosis

Patient is a 46  year old lady who presents with exertional shortness of breath, her NYHA functional class is 2–3. She denies orthopnea or PND.

16.1  Physical Examination On examination, she looks generally well, blood pressure 140/70 mmHg, heart rate was 70 beats per minute: irregularly irregular, respiratory rate 16/min. JVP is flat. Heart has normal S1, S2 with a diastolic rumble best heard in apical area. Lungs are clear. There is no adventitious sound. Both lower extremities are symmetric in size with no edema. Echocardiogram showed: 1. Normal LV size and LV systolic function. 2 . Rheumatic mitral valve disease with severe mitral valve stenosis (MS), (Fig. 16.1; Movies 16.1 and 16.2) mitral valve area is 0.85  cm2 by planimetry, mean gradient of mitral valve is 8 mmHg at the heart rate of 60 beats per minute

Electronic Supplementary Material The online version of this chapter (https://doi.org/10.1007/978-3-030-16618-2_16) contains supplementary material, which is available to authorized users.

© Springer Nature Switzerland AG 2019 A. Rohani, Clinical Cases in the Echocardiography Lab, Clinical Cases in Cardiology, https://doi.org/10.1007/978-3-030-16618-2_16

65

Chapter 16.  MV Stenosis

66

Figure 16.1 3D construction of severe MS, red arrow shows tiny orifice of the valve

(bpm) and percutaneous transvenous mitral commissurotomy (PTMC) score is 8, mild pulmonary hypertension with right ventricular systolic pressure (RVSP) of 47  mm and severely increased LA size.

16.2  Echo Pearls [1, 2] 1. Rheumatic MV disease is diagnosed by typical dome shape valve, commissural fusion in the parasternal short-axis view of TTE (the most important echocardiographic sign), restricted opening of both leaflets, subvalvular thickening, reduced E-F slope of the anterior mitral leaflet on M-Mode and presence of aortic or tricuspid valve involvement. Wilkins score has been described for the assessment of rate of success of balloon valvuloplasty; it has four parts: • • • •

Mobility of leaflets Sub-valvular Thickening Valvular Thickening Calcification

References

67

Generally speaking, normal MV is categorized as grade 0, and by increase in degree of thickening (valvular/sub-valvular), calcification or restriction of valve motion, score could go as high as 16 points. (Maximum point in each category is 4.)Score of more than 8 also is associated with lower rate of PTMC success. 2. Grading of stenosis: • Planimetry is the reference measurement for the assessment of severity of mitral valve stenosis; in para-­sternal short-axis view, investigate for the smallest orifice by scanning from apex to the base with lowest gain setting. MV area less than 1.5 cm2 is classified as severe stenosis. Pressure half-time>220 ms is also indicative of significant mitral valve stenosis, however it could be inaccurate in presence of atrial fibrillation with rapid ventricular response, severe aortic regurgitation (AR), and post-PTMC. • Mean gradient is easy to obtain but is dependent on blood flow and heart rate at the time of echocardiogram, mean gradient shouldn’t be used for grading of stenosis in the context of rheumatic valve involvement, it is also necessary to include heart rate in the report like this: (MV mean gradient is 5 mmHg at the heart rate of 74 bpm), another important key factor in the presence of atrial fibrillation is averaging at least five beats in all measurement.

References 1. Dr. Chi-Ming Chow. Mitral valvuloplasty score (MGH). [Online]. Available from: http://www.csecho.ca/wp-content/themes/twentyeleven-csecho/cardiomath/index.php?eqnHD=echo. 2. Baumgartner H, Hung J, Bermejo J, Chambers JB, Evangelista A, Griffin BP, Iung B, et al. CME test for echocardiographic assessment of valve stenosis: EAE/ASE recommendations for clinical practice. J Am Soc Echocardiogr. 2009;22(1):101–2.

Chapter 17 Bioprosthetic MV Partial Dehiscence

A 72 year old male with flail mitral valve underwent bioprosthetic mitral valve insertion 7 years ago. He developed progressive dyspnea, palpitations and presented with decompensated heart failure, examination revealed blood pressure of 138/72 mm Hg, grade 3/6 systolic murmur in apical area and bilateral pulmonary crackles. Patient was afebrile and did not have any clinical feature of infective endocarditis. Transthoracic echocardiography showed severe para- valvular leakage (PVL). Transesophageal echocardiography (TEE) demonstrated excessive rocking motion of the mitral bioprosthesis, eccentric MR jet and partial dehiscence (between 3 o’clock and 6 o’clock from surgical enface view) (Figs. 17.1 and 17.2; Movie 17.1).

Electronic Supplementary Material The online version of this chapter (https://doi.org/10.1007/978-3-030-16618-2_17) contains supplementary material, which is available to authorized users.

© Springer Nature Switzerland AG 2019 A. Rohani, Clinical Cases in the Echocardiography Lab, Clinical Cases in Cardiology, https://doi.org/10.1007/978-3-030-16618-2_17

69

70

Chapter 17.  Bioprosthetic MV Partial Dehiscence

Figure 17.1  3D construction partial dehiscence between 3–6 o’clock from surgical enface view (red arrows)

Figure 17.2  2D TEE 0 degree demonstrated eccentric paravalvular leakage

References

71

17.1  Echo Pearls [1–3] 1. For optimal result, increase the color frame rate and search for para-valvular leakage in several views and angles. 2. Identify extent of regurgitation as well as origin of the leakage. 3. Consider-three-dimensional echocardiography for determination of location and extension of leakage. 4. Dehiscence is identified by severe regurgitation and rocking motion of sewing ring with each cardiac cycle, especially with prosthetic aortic valve but rocking motion seen on the mitral side could be result of retained native posterior leaflet with no para-valvular regurgitation.

References 1. Koo HJ, Yang DH, Kang JW, Han K, Chung CH, Song JK, Lee I, et  al. Demonstration of prosthetic aortic valve dehiscence in a patient with noninfectious aortitis by multimodality imaging: findings of echocardiography and computed tomography. Circulation. 2013;128(7):759–61. 2. Ercan S, Altunbas G, Deniz H, Gokaslan G, Bosnak V, Kaplan M, et  al. Recurrent prosthetic mitral valve dehiscence due to ­infective endocarditis: discussion of possible causes. Korean J Thorac Cardiovasc Surg. 2013;46(4):285–8. 3. Rihal CS, Sorajja P, Booker JD, Hagler DJ, Cabalka AK. Principles of percutaneous paravalvular leak closure. JACC Cardiovasc Interv. 2012;5(2):121–30.

Chapter 18 Flail Posterior MV Leaflet in the Context of Rheumatic Heart Disease Patient was a 52 years old gentleman presented with progressive dyspnea. Examination revealed grade 3/6 systolic murmur in apical area and bi-basilar pulmonary crackles. Transthoracic echocardiography showed rheumatic mitral valve disease; thickening of anterior MV leaflet (AMVL) extends to sub-valvular chords and excessive leaflet tip motion of posterior mitral valve leaflet which leads to abnormal coaptation (Fig. 18.1 and Movie 18.1) and severe MR.

18.1  Echo Pearls [1–5] • To diagnose Rheumatic mitral valve disease, according to the World Heart Federation criteria, following echocardiographic features are important: 1. AMVL and chordal thickening, restricted leaflet motion (posterior leaflet is relatively immobile) and excessive leaflet tip motion during systole which can cause leaflet malcoaptation and regurgitation.

Electronic Supplementary Material The online version of this chapter (https://doi.org/10.1007/978-3-030-16618-2_18) contains supplementary material, which is available to authorized users.

© Springer Nature Switzerland AG 2019 A. Rohani, Clinical Cases in the Echocardiography Lab, Clinical Cases in Cardiology, https://doi.org/10.1007/978-3-030-16618-2_18

73

74

Chapter 18.  Flail Posterior MV Leaflet in the Context...

Figure 18.1  Red arrow shows thickened AMVL, Blue arrow shows flail gap and violet arrow shows flail posterior MV leaflet

• Definite Rheumatic Heart Disease (RHD) is defined as the presence of one of the following features in an individual more than 20 years old. –– Pathological MR and at least two morphological features of RHD of the MV. –– Mitral valve mean gradient ≥4 mmHg. –– Pathological AR and at least two morphological features of RHD of the AV, only in individuals aged 33 mm, inferior vena cava>21 mm, RV end-diastolic area >28 cm2 and maximal 2D RA volume >33 mL/m measured in apical 4 chamber TTE.)-Systolic flow reversal in hepatic vein. -Dense and triangular TR jet. -Vena Contracta>7mm.

References 1. Morokuma H, Minato N, Kamohara K, Minematsu N. Three surgical cases of isolated tricuspid valve infective endocarditis. Ann Thorac Cardiovasc Surg. 2010;16:134–8. 2. Habib G, Lancellotti P, Antunes MJ, Bongiorni MG, Casalta JP, Del Zotti F, et  al. 2015 ESC guidelines for the management of infective endocarditis: the task force for the management of infective endocarditis of the European Society of Cardiology (ESC). Endorsed by: European Association for Cardio-Thoracic Surgery (EACTS), the European Association of Nuclear Medicine (EANM). Eur Heart J. 2015;36(44):3075–128.

Chapter 24 Metallic Tricuspid Valve Obstruction

Patient is a 52 years old lady with prosthetic metallic tricuspid valve who presented with shortness of breath and lower extremities edema. She was non-compliant with warfarin in the past and recently she was off warfarin for a week. Two-­ dimensional color Doppler echocardiography revealed mean pressure gradient of 23.09 mmHg (Figs. 24.1 and 24.2; Movie 24.1). Fluoroscopy showed both cusps of tricuspid prosthesis got stuck in a partially open position.

24.1  Echo Pearls [1–3] Echocardiographic signs of prosthetic TV obstruction are as follows: • E velocity > 1.7 m/s. • Mean Pressure Gradient >6 mmHg. • PHT > 230 msec.

Electronic Supplementary Material The online version of this chapter (https://doi.org/10.1007/978-3-030-16618-2_24) contains supplementary material, which is available to authorized users.

© Springer Nature Switzerland AG 2019 A. Rohani, Clinical Cases in the Echocardiography Lab, Clinical Cases in Cardiology, https://doi.org/10.1007/978-3-030-16618-2_24

95

96

Chapter 24.  Metallic Tricuspid Valve Obstruction

Figure 24.1  TV in mid esophageal TEE short axis 44 degree

Figure 24.2  CW Doppler of TV shows MPG of 23.09 mmHg

References 1. Lancellotti P, Pibarot P, Chambers J, Edvardsen T, Delgado V, Dulgheru R, et  al. Recommendations for the imaging assessment of prosthetic heart valves: a report from the European Association of Cardiovascular Imaging endorsed by the

References

97

Chinese Society of Echocardiography, the Inter-American Society of Echocardiography, and the Brazilian Department of Cardiovascular Imaging. Eur Heart J Cardiovasc Imaging. 2016;17(6):589–90. 2. Gandhi N, Janssen M, Tavan E, Gold JA, Chadderdon SM.  Intermittent mechanical tricuspid valve obstruction recognized by central venous pressure tracing and confirmed by transesophageal echocardiography. J Am Coll Cardiol. 2012;60(10):e17. 3. Maragiannis D, Aggeli C, Nagueh SF. Echocardiographic evaluation of tricuspid prosthetic valves: an update. Hell J Cardiol. 2016;57(3):145–51.

Chapter 25 Bioprosthetic TV Obstruction

A 33-year-old woman admitted for progressive exertional shortness of breath, lower extremities edema and abdominal distention over the last couple of weeks. She underwent mitral and tricuspid valve replacement 2 years ago for rheumatic mitral and tricuspid valve stenosis. TTE and TEE revealed stenotic tricuspid valve prosthesis with the mean pressure gradient of 13.3  mmHg (Figs.  25.1, 25.2, and 25.3; Movie 25.1).

Electronic Supplementary Material The online version of this chapter (https://doi.org/10.1007/978-3-030-16618-2_25) contains supplementary material, which is available to authorized users.

© Springer Nature Switzerland AG 2019 A. Rohani, Clinical Cases in the Echocardiography Lab, Clinical Cases in Cardiology, https://doi.org/10.1007/978-3-030-16618-2_25

99

100

Chapter 25.  Bioprosthetic TV Obstruction

Figure 25.1  Doppler CW of TV shows MPG of 13.33 mmHg

Figure 25.2  Degenerative TV (red arrow) by 3D TEE

25  Bioprosthetic TV Obstruction

Figure 25.3  Degenerative TV (red arrow) by 2D TEE

101

Part IV

Pulmonic Valve (PV)

Chapter 26 Pulmonary Valve Stenosis

Patient was a 30 years old lady presented with dyspnea and fatigue over the past three months. New  York Heart Association class was III. She never smoked, and there was no family history of heart or lung disease. Oxygen saturation was 96% on breathing room air. There was a 3/6 systolic murmur heard loudest at the left upper sternal border. Electrocardiogram revealed right bundle branch block. Transthoracic echocardiogram findings include RV hypertrophy; (RVH) (Fig. 26.1), dilated pulmonary artery distal to the pulmonic valve and severe valvular PS (Fig. 26.2 and Movie 26.1).

26.1  Echo Pearls [1, 2] • Post-stenotic dilation of the pulmonary artery is due to weakness in the arterial wall and is not necessarily in proportion with degree of obstruction. • Pay attention to the morphology of pulmonary valve. • Grading of stenosis is illustrated in Table 26.1.

Electronic Supplementary Material The online version of this chapter (https://doi.org/10.1007/978-3-030-16618-2_26) contains supplementary material, which is available to authorized users.

© Springer Nature Switzerland AG 2019 A. Rohani, Clinical Cases in the Echocardiography Lab, Clinical Cases in Cardiology, https://doi.org/10.1007/978-3-030-16618-2_26

105

106

Chapter 26.  Pulmonary Valve Stenosis

Figure 26.1  Red arrow shows RVH para-sternal long axis, TTE

Figure 26.2  Thickened pulmonary valve (red arrow) Table 26.1  Echocardiographic characteristic of mild, moderate and severe PS Mild Moderate Severe Velocity m/s 4 Peak gradient mmHg

36

36–64

>64

References

107

• RVH: The parasternal long-axis and sub-costal long-axis views are often best views to assess thickness of RV wall; thickness more than 5 mm is usually considered abnormal. • Evaluate for atrial-level of shunting especially if RV is dilated.

References 1. Baumgartner H, Hung J, Bermejo J, Chambers J, Evangelista A, Griffin B, et  al. Echocardiographic assessment of valve stenosis: EAE/ASE recommendations for clinical practice. Eur J Echocardiogr. 2009;22(1):1–23. 2. Ayad RF, Johnston SB, Grayburn PA, Schmidt TT, Choi JW.  Congenital pulmonic stenosis in a 77-year-old woman successfully treated with percutaneous balloon valvuloplasty. Proc (Bayl Univ Med Cent). 2010;23(1):21–3.

Chapter 27 Sub-valvular Pulmonary Stenosis

A 23-year-old woman referred to cardiology service because of systolic murmur. On examination, blood pressure was 120/72  mmHg and the pulse rate was 70/min with a regular rhythm. Physical examinations revealed a harsh, grade 4/6 systolic ejection murmur, most prominent in the 4th intercostal space, left sternal border. ECG showed right ventricular hypertrophy. On echocardiography, there was a systolic jet in the right ventricular outflow tract (RVOT) proximal to the pulmonic valve (Figs.  27.1 and 27.2; Movie 27.1). Continuous wave Doppler revealed a mean pressure gradient (MPG) between the right ventricle and the distal portion of the right ventricular outflow tract of 83.6 mmHg and peak gradient of 160 mmHg (Fig. 27.3). Patient underwent surgical removal of a fibromuscular band obstructing the infundibulum; (infundibular pulmonary stenosis).

Electronic Supplementary Material The online version of this chapter (https://doi.org/10.1007/978-3-030-16618-2_27) contains supplementary material, which is available to authorized users.

© Springer Nature Switzerland AG 2019 A. Rohani, Clinical Cases in the Echocardiography Lab, Clinical Cases in Cardiology, https://doi.org/10.1007/978-3-030-16618-2_27

109

110

Chapter 27.  Sub-valvular Pulmonary Stenosis

Figure 27.1 TTE, short axis shows accelerated systolic flow in RVOT

Figure 27.2  Red arrow shows normal pulmonary valve, blue arrow shows subvalvular membrane (TEE 50 degree, mid esophageal)

References

111

Figure 27.3  Doppler in RVOT shows MPG of 83.6 mmHg

27.1  Echo Pearls for Subvalvular PS [1, 2] 1. Misdiagnosis of subvalvular PS as VSD or underestimation of disease severity is not uncommon. 2. Interrogate for the origin of jet to identify the exact location of stenosis.

References 1. Rahman MM, MSA S, Hasan MK, Adhikary AB.  Isolated Infundibular pulmonary stenosis in adult: a case report. J Dhaka Med Coll. 2014;23(1):140–2. 2. Shyu KG, Tseng CD, Chiu IS, Hung CR, Chu SH, Lue HC, et al. Infundibular pulmonic stenosis with intact ventricular septum: a report of 15 surgically corrected patients. Int J Cardiol. 1993;41:115–21.

Chapter 28 Supra-valvular Pulmonary Stenosis

A 36-year-old man referred for evaluation of exertional shortness of breath. Physical examination revealed blood pressure 120/70 mmHg and heart rate of 81beats/min. There was a 2/6 systolic ejection murmur over the left sternal border. Chest was clear. Electrocardiogram demonstrated right bundle branch block. Echocardiography showed normal right ventricle size and systolic function. Pulmonary valve looks normal, but there is flow acceleration inside right pulmonary artery (RPA) (Figs. 28.1 and 28.2, Movie 28.1). A transesophageal echocardiography confirmed a supravalvular membranous structure inside RPA with maximum pressure gradient of 62.86 mmHg (Figs. 28.3 and 28.4).

Electronic Supplementary Material The online version of this chapter (https://doi.org/10.1007/978-3-030-16618-2_28) contains supplementary material, which is available to authorized users.

© Springer Nature Switzerland AG 2019 A. Rohani, Clinical Cases in the Echocardiography Lab, Clinical Cases in Cardiology, https://doi.org/10.1007/978-3-030-16618-2_28

113

114

Chapter 28.  Supra-valvular Pulmonary Stenosis

Figure 28.1  Flow acceleration inside right pulmonary artery

Figure 28.2  Narrowing inside RPA, TTE short axis of great vessel

Chapter 28.  Supra-valvular Pulmonary Stenosis

115

Figure 28.3  TEE 0 degree high esophageal view shows flow acceleration inside RPA

Figure 28.4  Maximum pressure gradient of 62.86 mmHg in RPA

116

Chapter 28.  Supra-valvular Pulmonary Stenosis

28.1  Echo Pearls [1, 2] There is not a specific severity grading scale for branch PA stenosis, but if patient is symptomatic, repair should be considered.

References 1. Maffè S, Dellavesa P, Paffoni P, Zenone F, Cucchi L, Paino AM, et al. Isolated supravalvular pulmonary stenosis: a rare echocardiographic finding. Echocardiography. 2014;31(17):E215–7. 2. Habash S, Haas NA, Laser KT. Interventional therapy of supravalvular pulmonary stenosis via a mechanical valve in the pulmonary position. Congenit Heart Dis. 2014;9(2):E41–5.

Chapter 29 Severe Pulmonary Insufficiency (PI)

Patient was a 33  years old man, presented with worsening shortness of breath over the last year. He had palpitation and dizziness. He was diagnosed with Tetralogy Of Fallot (TOF) and underwent surgical repair during infancy. On examination he was not in respiratory distress, pulse rate 78 bpm, respiratory rate 18 /min, and blood pressure 119/73 mmHg. There was no central cyanosis. Heart examination revealed normal S1, S2 and grade 2/6 diastolic murmur in second left intercostals space. TTE showed severely increased right ventricular size, PRi (pulmonary regurgitation index = 0.78) and severe pulmonary valve regurgitation (Figs.  29.1, 29.2, and 29.3; Movie 29.1).

Electronic Supplementary Material The online version of this chapter (https://doi.org/10.1007/978-3-030-16618-2_29) contains supplementary material, which is available to authorized users.

© Springer Nature Switzerland AG 2019 A. Rohani, Clinical Cases in the Echocardiography Lab, Clinical Cases in Cardiology, https://doi.org/10.1007/978-3-030-16618-2_29

117

118

Chapter 29.  Severe Pulmonary Insufficiency (PI)

Figure 29.1 Apical 4 chamber shows severe right ventricular enlargement

Figure 29.2  Shows severe RV enlargement in parasternal long axis view, TTE

29.1  Echo Pearls

119

Figure 29.3  Severe PI CW Doppler

29.1  Echo Pearls 29.1.1  Severe Pi Echocardiographic Signs [1–3] • The ratio of PR jet width divided by RV outflow diameter: Mild ≤1/3; moderate 1/3–2/3; and severe ≥2/3 • PR index (Pri): Ratio of duration of PR divided by duration of diastole more than 0.77. • Pressure half time of regurgitation jet less than 100 ms. • Presence of diastolic flow reversal in branch pulmonary arteries. • Dilated RV size (RV end diastolic area index less than 20  cm2/m2 had 100% specificity to predict indexed RV volume ≤170 mL/m2 in cardiac MRI. • Continuous wave Doppler (CW) of regurgitation jet shows dense, steep deceleration and early termination of diastolic flow.

120

Chapter 29.  Severe Pulmonary Insufficiency (PI)

References 1. Zoghbi W, Adams D, Bonow R, Enriquez-Sarano M, Foster E, Grayburn P, et  al. Recommendations for noninvasive evaluation of native valvular regurgitation a report from the American Society of Echocardiography developed in collaboration with the Society for Cardiovascular Magnetic Resonance. J Am Soc Echocardiogr. 2017;30(4):303–71. 2. Yang H, Pu M, Chambers CE, Weber HS, Myers JL, Davidson WR Jr. Quantitative assessment of pulmonary insufficiency by Doppler echocardiography in patients with adult congenital heart disease. J Am Soc Echocardiogr. 2008;21(2):157–64. 3. Jhaveri RR, Saric M, Kronzon I. Uncommon Doppler echocardiographic findings of severe pulmonic insufficiency. J Am Soc Echocardiogr. 2010;23(10):1071–5.

Chapter 30 Pulmonary Valve Endocarditis

A 72-year-old male with a history of cancer presented to the hospital with increased shortness of breath and weight loss after 4  weeks treatment with IV antibiotic (due to MV endocarditis) administered through peripherally inserted central catheter (PICC). His TEE confirmed vegetations on mitral valve and PV. (Fig. 30.1). Patient had successful mitral and pulmonary valve replacements with tissue valves.

30.1  Echo Pearls [1, 2] • Pulmonary valve endocarditis is rare, but it is important to check this forgotten valve, as part of a comprehensive TEE exam, especially in a patient with a PICC line. • Pulmonary valve could be assessed by TEE mid esophageal view 45–60 degree and high esophageal view, 0 degree with right ward rotation of probe: first start with apical 4

© Springer Nature Switzerland AG 2019 A. Rohani, Clinical Cases in the Echocardiography Lab, Clinical Cases in Cardiology, https://doi.org/10.1007/978-3-030-16618-2_30

121

122

Chapter 30.  Pulmonary Valve Endocarditis

Figure 30.1  Pulmonary valve endocarditis (red arrow) mid-esophageal 88 degrees

chamber view on 0 degree, then pull out the probe: pulmonary valve can be visualized beside ascending aorta. (Fig.  30.2), sometimes increasing the angle will help to better visualize the valve, this view also is also very helpful for obtaining gradient across the pulmonary valve.

References

123

Figure 30.2  High esophageal view of ascending aorta and pulmonary valve

References 1. Schaefer A, Meyer G, Waldow A, Weiss T, Hausmann D, Drexler H. Pulmonary valve endocarditis. Circulation. 2001;103(9):e53–4. 2. Vrettos A, Mota P, Nash J, Thorp I, Baghai M, Baghai M. Pneumococcal pulmonary valve endocarditis. Echo Res Pract. 2017;4(3):K1–5.

Part V

Left Ventricle (LV)

Chapter 31 Non-compaction Cardiomyopathy

Patient is a young 23  year old lady who presented with atypical chest pain and palpitations. Her physical examination was unremarkable. She underwent echocardiography which revealed heavy trabeculation in left ventricle apex with visualization of blood flow into the inter-trabecular recesses. The ratio of non-compact myocardium to compact myocardium at the end of systole was >2:1 (Fig. 31.1; Movies 31.1 and 31.2) which fulfils Jenni criteria [1] for diagnosis of LV non-­compaction cardiomyopathy.

31.1  Echo Pearls [1–3] • Technical consideration: Consider positioning of focus in the apical region and if technically difficult, use intravenous contrast agents for better visualization of deep intertrabecular recesses.

Electronic Supplementary Material The online version of this chapter (https://doi.org/10.1007/978-3-030-16618-2_31) contains supplementary material, which is available to authorized users.

© Springer Nature Switzerland AG 2019 A. Rohani, Clinical Cases in the Echocardiography Lab, Clinical Cases in Cardiology, https://doi.org/10.1007/978-3-030-16618-2_31

127

128

Chapter 31.  Non-compaction Cardiomyopathy

Figure 31.1 Non-compact to compact ratio  =  3.6/1 LV short axis TTE at the end-systole

• Non-compaction cardiomyopathy diagnostic criteria by TTE are as follows: 1. Perfusion of deep inter-trabecular recesses in the myocardium identified by color Doppler flow, typical location is in LV apex and mid-inferior and lateral wall. 2. The basal and mid-interventricular septum is typically free of non-compaction. • Non-compaction-to-compaction ratio >2.0 at end-systole on LV short-axis parasternal view. • Absence of other cardiac abnormalities.

References

129

References 1. Jenni R, Oechslin E, Schneider J, Attenhofer Jost C, Kaufmann PA.  Echocardiographic and pathoanatomical characteristics of isolated left ventricular non-compaction: a step towards classification as a distinct cardiomyopathy. Heart. 2001;86(6):666–71. 2. Saleeb S, Margossian R, Spencer C, Alexander M, Smoot L, Dorfman A, et al. Reproducibility of echocardiographic ­diagnosis of left ventricular noncompaction. J Am Soc Echocardiogr. 2012;25(2):194–202. 3. Stöllberger C, Finsterer J.  Left ventricular hypertrabeculation/ noncompaction. J Am Soc Echocardiogr. 2004;17(1):91–100.

Chapter 32 Twenty Years After Hydatid Cyst Operation

Patient is a 55 year old lady with previous history of cardiac hydatid cyst surgery 20 years ago. She complains of shortness of breath with exertion; she can only walk one flight of stairs without dyspnea. She denies orthopnea, PND, chest pain, dizziness or syncope. On examination, JVP is flat; there is a harsh apical systyolic murmur grade 3/6. Chest is clear with no adventitious sound. 2D TTE shows a ruptured hydatid cyst hollowed on the right side of the inter-ventricular septum, drains to the right ventricle and no connection to the left ventricle. She has right ventricular systolic pressure of 60 mmHg which is consistent with moderate pulmonary hypertension (Figs. 32.1 and 32.2; Movie 32.1). This patient also suffers from rheumatic heart disease; she has rheumatic mitral and aortic valve disease; typical dome shape MV, mild mitral stenosis and mild to moderate aortic regurgitation.

Electronic Supplementary Material The online version of this chapter (https://doi.org/10.1007/978-3-030-16618-2_32) contains supplementary material, which is available to authorized users.

© Springer Nature Switzerland AG 2019 A. Rohani, Clinical Cases in the Echocardiography Lab, Clinical Cases in Cardiology, https://doi.org/10.1007/978-3-030-16618-2_32

131

132

Chapter 32.  Twenty Years After Hydatid Cyst Operation

Figure 32.1 Para sternal view shows connection between septum and RV

Patient underwent chest CT scan, which showed multiple small distal branch pulmonary embolism and a small area of pulmonary infarction. Casoni and Weinberg serologic test for hydatid cyst were positive. Because of multiple comorbidites, we decide to manage her medically.

32.1  Echo Pearls [1, 2] 1. The most common site for cardiac hydatid cyst disease is left ventricle. 2. This perforated cyst was very interesting but at the same time, there was rheumatic involvement of MV and AV which could be easily missed.

References

133

Figure 32.2  Ruptured hydatid cyst

References 1. Bayezid O, Ocal A, Isik O, Okay T, Yakut C.  A case of cardiac hydatid cyst localized on the interventricular septum and causing pulmonary emboli. J Cardiovasc Surg. 1991;32(3):324–6. 2. Özer N, Aytemir K, Kuru G, Atalar E, Özer Y, Övünc K, et  al. Hydatid cyst of the heart as a rare cause of embolization: report of 5 cases and review of published reports. J Am Soc Echocardiogr. 2001;14:299–302.

Chapter 33 Amyloid Heart Disease

A 77-year-old male with history of diabetes, hypertension and dyslipidemia presented with shortness of breath on exertion, and lower extremities edema. The onset of dyspnea was gradual and progressive; he denied chest pain. Physical examination revealed normal BP despite discontinuation of his medications, irregular heart rhythm, a faint systolic murmur best heard in the apex, elevated jugular venous pressure, fine bibasilar crackles and 1+ pitting edema bilaterally. Past medical history was prominent for bilateral carpal tunnel syndrome. Two dimensional echocardiography showed markedly thickened and speckled appearance of myocardium, bi-atrial enlargement, left ventricular ejection fraction (LVEF) of 49%, thickened atrio-ventricular valves, small pericardial effusion and diastolic dysfunction consistent with restrictive pattern and markedly elevated filling pressure (Figs.  33.1, 33.2, and 33.3; Movie 33.1). Rectal biopsy demonstrates transtyretin type amyloid which confirmed diagnosis of senile amyloidosis.

Electronic Supplementary Material  The online version of this chapter (https://doi.org/10.1007/978-3-030-16618-2_33) contains supplementary material, which is available to authorized users.

© Springer Nature Switzerland AG 2019 A. Rohani, Clinical Cases in the Echocardiography Lab, Clinical Cases in Cardiology, https://doi.org/10.1007/978-3-030-16618-2_33

135

136

Chapter 33.  Amyloid Heart Disease

Figure 33.1 Thickened inter atrial septum and valves, biatrial enlargement and pericardial effusion

Figure 33.2  Very short and steep deceleration time of mitral valve in pulse wave Doppler of MV

33.1  Echo Pearls

137

Figure 33.3  Markedly elevated E/Ea ratio

33.1  Echo Pearls [1–4] Echocardiographic features of amyloid heart disease are as follows: 1. Thickening of the left ventricular wall and low voltage ECG as well as concentric RV wall thickening. (QRS voltage amplitude 0.5 mV in all limb leads or 1 mV in all precordial leads) 2. Speckled or granular myocardial (sparkling) appearance (low sensitivity). (Be careful about gain setting) 3. Thickened inter-atrial septum (100% specificity in one report) 4. Bi-atria enlargement. 5. Thickened valves. 6. Pericardial effusion. 7. Diastolic dysfunction with low E` is the hallmark (restrictive pattern and markedly elevated filling pressure.) (Figs. 33.2 and 33.3)

138

Chapter 33.  Amyloid Heart Disease

8. Reduction of LV torsion and untwisting. 9. Relatively well preserved LV apex longitudinal strainx and reduced longitudinal strain at basal level is called apical sparing; it is a well-known echocardiographic sign of cardiac amyloidiosis, however this sign may not be presented in cardiac amyloidosis associated with aortic valve stenosis, which is not uncommon in older population [5]. 10. Atrial strain is also globally impaired.

References 1. Selvanayagam V, Hawkins P, Paul B.  Evaluation and management of the cardiac amyloidosis. J Am Coll Cardiol. 2007;50(22):2101–10. 2. Siqueira-Filho AG, Cunha CL, Tajik AJ, et al. M-mode and two-­ dimensional echocardiographic features in cardiac amyloidosis. Circulation. 1981;63(1):188–96. 3. Ebrille E, Di Donna P, Leuzzi S, Miceli S, Gaita F, Scaglione M.  Cardiac amyloidosis and hypertrophic cardiomyopathy: a dangerous liaison. Glob Cardiol Sci Pract. 2013;2013(4):405–8. 4. Park SJ, Miyazaki C, Bruce CJ, Ommen S, Miller FA, Oh JK. Left ventricular torsion by two-dimensional speckle tracking echocardiography in patients with diastolic dysfunction and normal ejection fraction. J Am Soc Echocardiogr. 2008;21(10):1129–37. 5. Phelan D, Collier P, Thavendiranathan P, Popovic ZB, Hanna M, Plana JC, et al. Relative apical sparing of longitudinal strain using two-dimensional speckle-tracking echocardiography is both sensitive and specific for the diagnosis of cardiac amyloidosis. Heart. 2012;98(19):1442–8.

Chapter 34 Apical Hypertrophic Cardiomyopathy (aHCM)

A 46  year old man presented with chest discomfort during exercise, he denied sweating, nausea, vomiting, and shortness of breath. There is no malignant family history for sudden cardiac death or premature coronary artery disease. On examination blood pressure was 125/72 mmHg, heart rate 67 bpm, regular, JVP is flat, chest is clear, heart has normal S1, S2 with no murmur. Both lower extremities are symmetric in size with no edema. Echocardiogram showed typical apical hypertrophic cardiomyopathy (Ace-of-Spades) and apical wall thickness ≥15 mm (Fig. 34.1 and Movie 34.1).

Electronic Supplementary Material The online version of this chapter (https://doi.org/10.1007/978-3-030-16618-2_34) contains supplementary material, which is available to authorized users.

© Springer Nature Switzerland AG 2019 A. Rohani, Clinical Cases in the Echocardiography Lab, Clinical Cases in Cardiology, https://doi.org/10.1007/978-3-030-16618-2_34

139

140

Chapter 34.  Apical Hypertrophic Cardiomyopathy...

Figure 34.1  Apical wall thickness = 20 mm, TTE 4 chamber apical view

34.1  Echo Pearls [1–3] 1. It is very important to have proper imaging of left ventricular apex; a thickened apex may be misinterpreted as foreshortened apex; intravenous contrast agents could improve diagnostic accuracy. • Another important key is Doppler recording of blood velocity at the apical, mid-ventricular and outflow tract level for rule out obstruction. • The diagnostic criteria for apical hypertrophic cardiomyopathy is hypertrophy predominantly at the LV apex with an apical wall thickness ≥15  mm and a ratio of maximal apical to posterior wall thickness ≥1.5 or more during diastole determined by two dimensional echocardiography or cardiac magnetic resonance imaging.

References

141

• Spade-like configuration of the left ventricular cavity at end-diastole on left ventriculography. 2. There are two sub- groups: • Isolatedasymmetricapicalhypertrophy—“pure”ApHCM— • Coexistent hypertrophy of the interventricular septum—“mixed” ApHCM (greatest wall thickness in the apical segments.)

References 1. Eriksson M, Sonnenberg B, Woo A, Rakowski P, Parker T, Wigle E, et al. Long-term outcome in patients with apical hypertrophic cardiomyopathy. J Am Coll Cardiol. 2002;39(4):638–45. 2. Stainback F. Apical hypertrophic cardiomyopathy. Tex Heart Inst J. 2012;39(5):747–9. 3. Chen CC, Lei MH, Hsu YC, Chung SL, Sung YJ.  Apical hypertrophic cardiomyopathy: correlations between echocardiographic parameters, angiographic left ventricular morphology, and clinical outcomes. Clin Cardiol. 2011;34:233–8.

Chapter 35 Hypertrophic Obstructive Cardiomyopathy

Patient was a 59  years old woman who was referred for assessment of systolic murmur; she had three echocardiographic assessments in the past which interpreted as mild aortic valve stenosis. This time she was found to have normal aortic valve opening but severe LVOT obstruction with peak gradient of 30  mmHg at rest and 52  mmHg with valsalva maneuver. As her TTE images were technically difficult, she underwent TEE which showed echocardiographic evidences of HOCM: • Systolic anterior motion of mitral valve – SAM (Fig. 35.1 and Movie 35.1). • Asymmetric LV hypertrophy (trans-gastric view, Fig. 35.2) • LVOT obstruction (Fig. 35.3)

Electronic Supplementary Material The online version of this chapter (https://doi.org/10.1007/978-3-030-16618-2_35) contains supplementary material, which is available to authorized users.

© Springer Nature Switzerland AG 2019 A. Rohani, Clinical Cases in the Echocardiography Lab, Clinical Cases in Cardiology, https://doi.org/10.1007/978-3-030-16618-2_35

143

144

Chapter 35.  Hypertrophic Obstructive Cardiomyopathy

Figure 35.1  Red arrow shows severe MR due to SAM, blue arrow shows systolic flow acceleration due to LVOT obstruction

Figure 35.2  Trans-gastric TEE 0 degree shows thickened septum

35.1  Echo Pearls

145

35.1  Echo Pearls [1, 2] • HCM is defined as unexplained maximal wall thickness >15  mm in any myocardial segment, or septal/posterior wall thickness ratio >1.3  in normotensive patients, or Septal/posterior wall thickness ratio >1.5  in hypertensive patients. Therefore, presence of mid cavitary gradient is not equivalent of HOCM, first asymmetrical hypertrophy must be present. • Please note that sometimes MR jet is contaminated with LVOT gradient, try to find dagger shape doppler wave. • Be very careful when interpret any mid cavitary gradient, always pay attention to the aortic valve morphology. As I presented here, this case was misinterpreted as aortic valve stenosis. There are also other causes of mid cavity gradient like as tachycardia, dehydration, small LV cavity and anomalous papillary muscle insertion (Figs. 35.4 and 35.5).

Figure 35.3  Typical dagger type CW doppler of LVOT obstruction

146

Chapter 35.  Hypertrophic Obstructive Cardiomyopathy

Figure 35.4 TTE apical 4 chamber view with LV contrast shows anomalous papillary muscle (red arrow) divided LV cavity to two chambers (green arrows) causing mid cavity gradient

Figure 35.5 TEE of anomalous papillary muscle, red arrow, long axis view, mid-esophageal

References

147

References 1. Rowin EJ, Maron BJ, Lesser JR, Rastegar H, Maron MS. Papillary muscle insertion directly into the anterior mitral leaflet in hypertrophic cardiomyopathy, its identification and cause of outflow obstruction by cardiac magnetic resonance imaging, and its surgical management. Am J Cardiol. 2013;111(11):1677–9. 2. Yang HS, Lee KS, Chaliki HP, Tazelaar HD, Lusk JL, Chandrasekaran K, et  al. Anomalous insertion of the papillary muscle causing left ventricular outflow obstruction: visualization by real-time three-dimensional echocardiography. Eur J Echocardiogr. 2008;9(6):855–60.

Chapter 36 Huge LV Apical Clot

A 62-year-old patient presented to the hospital with anterior ST elevation myocardial infarction. Transthoracic echocardiography (TTE) revealed anterior and apical wall akinesia with LV ejection fraction (EF) of 25%, no hemodynamically significant valve disease, and large mural (Fig.  36.1; Movie 36.1; Table  36.1) thrombus (10 cm × 12 cm) in the apex and distal anterior wall.

36.1  Echo Pearls [1–3] False tendons, trabecula, reverberations, side lobe or near field artifacts could be misdiagnosed as thrombus. • Intravenous echo contrast could improve diagnostic field. • 10–46% of echocardiograms are non diagnostic for LV thrombus assessment. • Table 36.1 classifies LV thrombus shape.

Electronic Supplementary Material The online version of this chapter (https://doi.org/10.1007/978-3-030-16618-2_36) contains supplementary material, which is available to authorized users.

© Springer Nature Switzerland AG 2019 A. Rohani, Clinical Cases in the Echocardiography Lab, Clinical Cases in Cardiology, https://doi.org/10.1007/978-3-030-16618-2_36

149

150

Chapter 36.  Huge LV Apical Clot

Figure 36.1  Big apical thrombus Table 36.1  Different LV thrombus shape Protruding Extends into the LV cavity Mural

Flat and parallel to endocardium

References 1. Srichai MB, Junor C, Rodriguez LL, et  al. Clinical, imaging, and pathological characteristics of left ventricular thrombus: a comparison of contrast-enhanced magnetic resonance imaging, transthoracic echocardiography, and transesophageal echocardiography with surgical or pathological validation. Am Heart J. 2006;152(1):75–84. 2. Delewi R, Zijlstra F, Piek JJ. Left ventricular thrombus formation after acute myocardial infarction. Heart. 2012;98:1743–9. 3. Weinsaft JW, Kim RJ, Ross M, Krauser D, Manoushagian S, LaBounty TM, et  al. Contrast-enhanced anatomic imaging as compared to contrast-enhanced tissue characterization for detection of left ventricular thrombus. JACC Cardiovasc Imaging. 2009;2(8):969–79.

Chapter 37 Source of Embolism to Brain

A 43-year-old woman presented with stroke. She developed left-sided weakness and aphasia. Echocardiography revealed large, lobulated and oscillating mass with myocardial texture inside LV apex (Fig.  37.1 and Movie 37.1).

37.1  Echo Pearls [1–3] • Care must be taken to use high-frequency and fundamental imaging (with sweeps) from multiple perspectives with and without harmonics. • Color Doppler with appropriate Nyquist shift is necessary for identifying any shunt (low shift for interatrial septal shunts and large VSDs, high shift for small VSDs); contrast study also is useful for determination of any shunt. • Echocardiographic characters of a mass should be reported as following: 1. Echogenicity/echo texture of mass. 2 . Size of mass: length and width. Electronic Supplementary Material The online version of this chapter (https://doi.org/10.1007/978-3-030-16618-2_37) contains supplementary material, which is available to authorized users.

© Springer Nature Switzerland AG 2019 A. Rohani, Clinical Cases in the Echocardiography Lab, Clinical Cases in Cardiology, https://doi.org/10.1007/978-3-030-16618-2_37

151

152

Chapter 37.  Source of Embolism to Brain

Figure 37.1  Red arrow shows apical mass (TEE trans-gastric view 97 degree)

3. Aspect/shape of the mass: amorphous, shaggy, lobulated, linear or round. 4. Location of mass: atrial side of atrioventricular valves, ventricular side of the aortic valve. 5. Motion of the mass: high-frequency flutters/oscillating/ chaotic/orbiting/independent or dependent to the valve motion; if mass is large enough, it may prolapse into ventricles in diastole.

References 1. Yoo M, Graybeal DF.  An echocardiographic-confirmed case of atrial myxoma causing cerebral embolic ischemic stroke: a case report. Cases J. 2008;1(1):96. 2. Pinede L, Duhaut P, Loire R. Clinical presentation of left atrial cardiac myxoma. A series of 112 consecutive cases. Medicine (Baltimore). 2001;80(3):159–72. 3. Abbasi AS, Costa MD, Hennessy T, Kiernan TJ. Cardiac papillary fibroelastoma presenting as acute stroke. BMJ Case Rep. 2013;2013:bcr2013010092.

Chapter 38 Huge Calcified Mass, Inside Heart or Outside?

Patient is an 80  year old gentleman known for multiple endocrine neoplasia type2 (MEN2) syndrome, admitted to the hospital because of hypercalcemia. Past medical history is prominent for medullary thyroid cancer, parathyroid tumor and acromegalia. On admission, clinical examination revealed stable vital sign, irregular heart rhythm and a soft systolic apical murmur grade 2/6. The ECG showed atrial fibrillation with the heart rate of 87 bpm. Laboratory investigation revealed impaired renal function and hypercalcemia. Two dimensional trans- thoracic and transesophageal echocardiography (2D TTE, TEE) demonstrated preserved left ventricle systolic function and an echogenic mass which seems to be inside the left atrium (Fig. 38.1 and Movie 38.1). Real time 3D TEE (Movie 38.2) provides additional information about the origin of mass. Enface view of MV Electronic Supplementary Material The online version of this chapter (https://doi.org/10.1007/978-3-030-16618-2_38) contains supplementary material, which is available to authorized users.

© Springer Nature Switzerland AG 2019 A. Rohani, Clinical Cases in the Echocardiography Lab, Clinical Cases in Cardiology, https://doi.org/10.1007/978-3-030-16618-2_38

153

154

Chapter 38.  Huge Calcified Mass, Inside Heart or Outside?

Figure 38.1  Intra or extra cardiac mass seen in TEE mid-esophageal 0 degree

Figure 38.2  Extra cardiac mass with pressure effect on LA seen on 3D mid esophageal TEE

References

155

showed a fixed extra cardiac mass (Fig. 38.2) confirmed by surgical excision.

38.1  Echo Pearls [1–3] This case highlights clinical advantage of three-dimensional echocardiography in a case of multiple endocrine neoplasia type 2 (MEN2) syndrome for assessment of cardiac mass. Cardiac mass is a particular clinical challenge, and only pathologic study could provide the final diagnosis. Three-­ dimensional echocardiography is a useful tool for the evaluation and determination of location of a mass.

References 1. Asch FM, Bieganski SP, Panza JA, Weissman NJ.  Real-time 3-dimensional echocardiography evaluation of intracardiac masses. Echocardiography. 2006;23(3):218–24. 2. Lokhandwala J, Zheng L, Jundi M, Loyd M, Strong M, Vannan M. Three-dimensional echocardiography of intracardiac masses. Echocardiography. 2004;21(2):159–63. 3. Papadopoulos CH, Michalakeas CA, Paraskevaidis I, Ikonomidis I, Anastasiou-Nana M. Differential diagnosis of a left atrial mass: role of three-dimensional transoesophageal echocardiography. Hell J Cardiol. 2010;51(6):546–8.

Part VI

Congenital Heart Disease

Chapter 39 Coarctation of Aorta (Co-A)

Patient is a 26 year old young man who presents with fatigue. He had left arm blood pressure of 180/90  mmHg, right arm pressure of 180/90  mmHg and lower extremity pressure of 90 mmHg by Doppler study. A systolic murmur heard in the left infra-clavicular area, and an ejection click in aortic area. Electrocardiography revealed left ventricular hypertrophy. TTE demonstrated bicuspid aortic valve, mild aortic regurgitation, no aortic stenosis and significant coarctation of aorta after left subclavian artery (systolic velocity of 3.5  m/s, diastolic velocity of 2 m/s and ratio of diastolic velocity at the end of T wave on ECG divided by systolic velocity = 0.57). Real time 3D echocardiography clearly demonstrated a discrete shelf-like structure with an eccentric opening inside thoracic aorta (Figs. 39.1, 39.2, and 39.3; Movies 39.1 and 39.2).

Electronic Supplementary Material The online version of this chapter (https://doi.org/10.1007/978-3-030-16618-2_39) contains supplementary material, which is available to authorized users.

© Springer Nature Switzerland AG 2019 A. Rohani, Clinical Cases in the Echocardiography Lab, Clinical Cases in Cardiology, https://doi.org/10.1007/978-3-030-16618-2_39

159

160

Chapter 39.  Coarctation of Aorta (Co-A)

Figure 39.1 Prominent diastolic tale, red arrow shows diastolic velocity at the end of T wave

Figure 39.2  Red arrow shows systolic turbulent flow

39.1  Echo Pearls

161

Figure 39.3 3D construction of membranous obstruction of desending thoracic aorta

39.1  Echo Pearls [1, 2] • Bicuspid aortic valve (prevalence is up to 85%) and ventricular septal defect (VSD) are associated anomaly for coarctation of aorta. • Increased velocity across coarctation in descending aorta, described as “Shark-tooth” pattern; it is a systolic peak and slowly decremented diastolic taper of the Doppler envelope, related to diastolic “run-off” in the collateral circulation. The diastolic velocity at the end of T wave on ECG divided by systolic peak velocity more than 0.53 had a sensitivity of 100% and specificity of 96% for detecting severe Co-A. • Continuous flow in abdominal aorta demonstrates presence and extent of the collateral circulation. After coarctation repair, continuous flow may return to a normal pulsatile form.

162

Chapter 39.  Coarctation of Aorta (Co-A)

References 1. Warnes CA, Williams RG, Bashore TM, Child JS, Connolly HM, Dearani JA, et al. ACC/AHA 2008 guidelines for the management of adults with congenital heart disease: a report of the American College of Cardiology/American Heart Association task force on practice guidelines (writing committee to develop guidelines on the management of adults with congenital heart disease). Developed in collaboration with the American Society of Echocardiography, Heart Rhythm Society, International Society for Adult Congenital Heart Disease, Society for Cardiovascular Angiography and Interventions, and Society of Thoracic Surgeons. J Am Coll Cardiol. 2008;52(23):e143–263. 2. Tan JL, Babu-Narayan SV, Henein MY, Mullen M, Li W. Doppler echocardiographic profile and indexes in the evaluation of aortic coarctation in patients before and after stenting. J Am Coll Cardiol. 2005;46(6):1045–53.

Chapter 40 Patent Ductus Arteriosus Endocarditis

Patient is a 31 years old lady who presents to the hospital with fever, chills and shortness of breath; she has methicillin sensitive staph aureous bacterimia. She has a continuous murmur, best heard in left upper chest, wide plus pressure and prominent suprasternal pulsation. Echocardiogram showed large patent ductus arteriosus, [PDA] with mobile vegetation on the pulmonary side (Figs. 40.1, 40.2, and 40.3; Movie 40.1).

40.1  Echo Pearls [1, 2] 1. In the report of transthoracic echocardiogram of PDA following data must be reported: • Size of PDA. • Qp/Qs (Qp = Pulmonary flow. Qs = Systemic flow. Qp / Qs demonstrates the degree of a shunt. Normally = 1:1. Left to right shunts >1.0.) Electronic Supplementary Material The online version of this chapter (https://doi.org/10.1007/978-3-030-16618-2_40) contains supplementary material, which is available to authorized users.

© Springer Nature Switzerland AG 2019 A. Rohani, Clinical Cases in the Echocardiography Lab, Clinical Cases in Cardiology, https://doi.org/10.1007/978-3-030-16618-2_40

163

164

Chapter 40.  Patent Ductus Arteriosus Endocarditis

Figure 40.1  TTE PSAX view, red arrow shows PDA

Figure 40.2  Three dimensional construction of PDA. Red arrow is the aortic opening of PDA

• Left atrium size. • Size of aortic entrance and pulmonary entrance. (PDA is a duct between descending aorta and left pulmonary artery (LPA)).

40.1  Echo Pearls

165

Figure 40.3  Vegetation in the aortic opening site of PDA

• Duct peak systolic velocity and pressure gradient. • Shape of PDA. 1. Krichenko Type A: Most common (65%); funnel-shaped ductus with a localized narrowing at the pulmonary artery junction. 2. Krichenko Type B: (18%); funnel-shaped PDA with an aortic ampulla. 3. Krichenko Type C: Tubular shape. 4. Krichenko Type D: Oval shape with both aortic and pulmonary ampulla. 5. Krichenko Type E: Other anomalous forms. 2. How Transesophageal echocardiography can confirm presence of a PDA? (Movie 40.2) First identify the descending thoracic aorta, then slightly pull out the probe, you could see the PDA, and afterwards increase the angle to 60 degree, at this point PDA tunnel will show up.

166

Chapter 40.  Patent Ductus Arteriosus Endocarditis

References 1. Eerola A, Jokinen E, Boldt T, Pihkala J.  The influence of percutaneous closure of patent ductus arteriosus on left ventricular size and function: a prospective study using twoand three-­dimensional echocardiography and measurements of serum natriuretic peptides. J Am Coll Cardiol. 2006;47(5):1060–6. 2. Krichenko A, Benson LN, Burrows P, Möes CA, McLaughlin P, Freedom RM.  Angiographic classification of the isolated, persistently patent ductus arteriosus and implications for percutaneous catheter occlusion. Am J Cardiol. 1989;63(12):877–80.

Chapter 41 Secundum Atrial Septal Defect (ASD)

Patient is an 18-year-old young man with shortness of breath; he has dyspnea for 2–3 years which worsened over last 6 months. On examination, there is a soft mid systolic murmur on left lower sternal border with fixed splitting of S2. ECG showed normal sinus rhythm and right bundle branch block. 2D TTE showed dilated right ventricle and normal left and right ventricular systolic function.TEE showed a large ASD secundum (Figs. 41.1, 41.2, 41.3, 41.4, and 41.5; Movies 41.1 and 41.2).

41.1  Echo Pearls 1. Three-dimensional TEE (Movie 41.1) images could describe various shapes (round, oval) and size of ostium secundum ASD [1, 2]. 2. Measurement of all ASD rim is crucial for decision making of treatment plan (Figs. 41.2, 41.3, and 41.4; Movie 41.2).

Electronic Supplementary Material The online version of this chapter (https://doi.org/10.1007/978-3-030-16618-2_41) contains supplementary material, which is available to authorized users.

© Springer Nature Switzerland AG 2019 A. Rohani, Clinical Cases in the Echocardiography Lab, Clinical Cases in Cardiology, https://doi.org/10.1007/978-3-030-16618-2_41

167

168

Chapter 41.  Secundum Atrial Septal Defect (ASD)

Figure 41.1  Three-dimensional TEE of an oval shaped ASD

Figure 41.2  IVC and SVC rim best measured at 110 degree TEE bi-­caval view

41.1  Echo Pearls

Figure 41.3  Triangle ASD, 3D construction

Figure 41.4  Antero- inferior rim apical 4 chamber TEE

169

170

Chapter 41.  Secundum Atrial Septal Defect (ASD)

Figure 41.5  TEE bi-caval view shows ASD shunt

3. Always check for the presence of all pulmonary veins or any anomalous pulmonary vein.

References 1. Warnes CA, Williams RG, Bashore TM, et  al. ACC/AHA 2008 guidelines for the management of adults with congenital heart disease: a report of the American College of Cardiology/American Heart Association task force on practice guidelines (writing committee to develop guidelines on the management of adults with congenital heart disease). Developed in collaboration with the American Society of Echocardiography, Heart Rhythm Society, International Society for Adult Congenital Heart Disease, Society for Cardiovascular Angiography and Interventions, and Society of Thoracic Surgeons. J Am Coll Cardiol. 2008;52(23):143–263. 2. Baumgartner H, Bonhoeffer P, De Groot NM, et al. Task force on the management of grown-up congenital heart disease of the European Society of Cardiology (ESC); Association for European Paediatric Cardiology (AEPC); ESC Committee for Practice Guidelines (CPG). ESC guidelines for the management of grown-up congenital heart disease (new version 2010). Eur Heart J. 2010;31(23):2915–57.

Chapter 42 Atrioventricular Septal Defect (AVSD)

A 26-year-old woman presented with worsening exertional shortness of breath. On examination, she had a grade 3/6 systolic murmur, audible on the left para-sternal border. A transthoracic echocardiogram showed a large ostium primum atrial septal defect, a large inter-ventricular septal defect, and a common atrio-ventricular (AV) orifice (Fig. 42.1 and Movie 42.1). Doppler study revealed severe regurgitation of the common AV valve.

42.1  Echo Pearls [1, 2] • Table 42.1 describes complete, partial and transition form of AVSD. • Complete AVSD has three Rastelli subtypes according to the morphology of anterior bridging leaflet and its attachment: (Table 42.2 summarizes these subtypes) Electronic Supplementary Material The online version of this chapter (https://doi.org/10.1007/978-3-030-16618-2_42) contains supplementary material, which is available to authorized users.

© Springer Nature Switzerland AG 2019 A. Rohani, Clinical Cases in the Echocardiography Lab, Clinical Cases in Cardiology, https://doi.org/10.1007/978-3-030-16618-2_42

171

172

Chapter 42.  Atrioventricular Septal Defect (AVSD)

Figure 42.1  Complete AV canal, type A Rastelli

Table 42.1  AVSD types

Partial

Primum ASD Inlet VSD Present Absent

Common AV valve Absent

A single Intermediate Present Small interventricular valvular AVSD communication annulus: (transitional anterior and common posterior atrioventricular bridging canal) leaflets with two distinct valvular leaflet present. Complete Present Present atrioventricular canal

Present

The most frequently associated abnormality Cleft of the anterior mitral valve leaflet.

References

173

Table 42.2  Rastelli subtypes

Type A valve

Anterior leaflet Common atrioventricular valve has been divided to two equal size.

Medial Chordae tendineae attachment Attached to the crest of ventricular septal defect.

Type B valve

Anterior common AV leaflet is divided with right dominance.

Mitral and tricuspid part is both attached medially to anomalous papillary muscle positioned in the right ventricle.

Type C valve

Common AV leaflet is not divided

Free floating valve. No attachment.

• Both atrioventricular valves are at the same level. • The left ventricular outflow tract is displaced anteriorly, creates the characteristic “gooseneck” deformity seen on LV angiography.

References 1. Weale J, Kelleher AA. Adult congenital heart disease. Anaesthesia Intensive Care Med. 2015;16(10):528–34. 2. Huggon IC, Cook AC, Smeeton NC, Magee AG, Sharland GK.  Atrioventricular septal defects diagnosed in fetal life: associated cardiac and extra-cardiac abnormalities and outcome. J Am Coll Cardiol. 2000;36(2):593–601.

Chapter 43 L-Transposition of Great Arteries (L-TGA)

A 25-year-old male presented with exertional shortness of breath. He had no remarkable past medical or surgical history. On admission, blood pressure was 136/82  mm Hg and grade 2 systolic murmur was audible in the pulmonic area. Transthoracic echocardiography demonstrated coarse trabeculations of functional LV and fine trabeculations and smooth surfaced functional RV, which correspond with Levo-­ Transposition of the great arteries (L-Transposition of the great arteries/ L-TGA.) Figures  43.1 and 43.2; Movies 43.1 and 43.2. Left-sided systemic AV valve was displaced apically, which is consistent with anatomic tricuspid valve.

Electronic Supplementary Material The online version of this chapter (https://doi.org/10.1007/978-3-030-16618-2_43) contains supplementary material, which is available to authorized users.

© Springer Nature Switzerland AG 2019 A. Rohani, Clinical Cases in the Echocardiography Lab, Clinical Cases in Cardiology, https://doi.org/10.1007/978-3-030-16618-2_43

175

Figure 43.1  LTGA anatomic and physiologic illustration

Anatomic LV and MV Acting as physiologic RV

PV

AV

SVC

Anatomic RV and TV Acting as physiologic LV

Pulmonary artery

176 Chapter 43.  L-Transposition of Great Arteries (L-TGA)

43.1  Echo Pearls for Congenital Heart Disease...

177

Figure 43.2  Para-sternal long axis of the patient with LTGA clearly shows two parallel great arteries

Table 43.1  Define Atrial arrangement according to these findings Right atrium Left atrium Broad-based appendage Finger- or hook-shaped appendage with pectinate muscle. Inferior vena cava (IVC)/ Eustachian valve connection Crista terminalis

43.1  E  cho Pearls for Congenital Heart Disease [1, 2]; Step by Step Echocardiographic Approach • Tables 43.1 and 43.2 describe atrial and ventricular characteristic of right and left side chambers.

178

Chapter 43.  L-Transposition of Great Arteries (L-TGA)

Table 43.2  Define Ventricular Chambers Left ventricle Right ventricle Presence of one or Presence of three or several papillary two papillary muscle. muscle (one of which inserts into the interventricular septum) Tricuspid valve is more apical. Fine apical trabeculation

Coarse apical trabeculation, moderator band

More conical shape

Triangular shape

43.2  Next Steps • Define Arterial Trunks –– Main pulmonary artery bifurcates into the right and left branch, –– Aorta has no branch before aortic arch. • Define Atrio-ventricular Connections:

And

Ventriculo-Arterial

A. Concordant vs. Discordant –– Concordant atrioventricular (AV) connection: RA is connected to RV and LA to LV. –– Discordant AV connection: RA is connected to LV and LA is connected to RV. B. Overriding of the AV valves: –– 50% of inlet valve opens into the contra-lateral ventricle). –– Straddling of the AV valve: presence of chordal attachment crossing through an inter-ventricular septal defect and inserting into the contra-lateral ventricle.

References

179

C. Overriding of an arterial trunk: –– 50% an arterial trunk connects into the contra-­ lateral ventricle. –– Double outlet right ventricle (DORV) means more than 50% of aorta connects to RV.

43.3  LTGA • The absence of mitral–aortic continuity. • Both great vessels are parallel seen in para- sternal long axis view. • Left side atrio-ventricular valve is more apical than the right side.

References 1. Weale J, Kelleher AA. Adult congenital heart disease. Anaesthesia Intensive Care Med. 2015;16(10):528–34. 2. Park SH, Choi JY, Park EJ, Kim HD, Choi MJ, Choi SI, et al. A typical case of L-transposition of the great arteries initially presented as complete atrioventricular block in middle-aged man. J Cardiovasc Ultrasound. 2015;23(1):36–9.

Chapter 44 Sinus Venosus ASD

A 32–year–old male presented with exertional dyspnea and palpitations. Physical examination revealed a 2/6 systolic murmur at the left border of the sternum which increased with inspiration. ECG showed normal sinus rhythm and right bundle branch block. TEE showed sinus venosus ASD and anomalous connection of right upper pulmonary vein to RA (Figs.  44.1, 44.2, 44.3, and 44.4; Movie 44.1)

44.1  Echo Pearls [1, 2] Bicaval view must be interrogated in both mid and high esophageal view to visualize the superior region of inter atrial septum.

Electronic Supplementary Material The online version of this chapter (https://doi.org/10.1007/978-3-030-16618-2_44) contains supplementary material, which is available to authorized users.

© Springer Nature Switzerland AG 2019 A. Rohani, Clinical Cases in the Echocardiography Lab, Clinical Cases in Cardiology, https://doi.org/10.1007/978-3-030-16618-2_44

181

182

Chapter 44.  Sinus Venosus ASD

Figure 44.1  Sinus venosus ASD color Doppler (red arrow), TEE bi-­caval view

Figure 44.2  Sinus venosus ASD, TEE bi-caval view

44.1  Echo Pearls

183

Figure 44.3 Abnormal pulmonary vein Doppler flow below the baseline

Figure 44.4  Blue color shows abnormal right upper pulmonary vein connection

184

Chapter 44.  Sinus Venosus ASD

References 1. Oliver JM.  Sinus venosus syndrome: atrial septal defect or anomalous venous connection? A multiplane transoesophageal approach. Heart. 2002;88(6):634–8. 2. Sharma AK, Nath RK, Pandit N. A case of sinus venosus atrial septal defect misdiagnosed as primary pulmonary hypertension. Hell J Cardiol. 2016;57(2):124–8.

Chapter 45 Left Anomalous Pulmonary Vein Connection

Patient was a 31 years old woman who presented with shortness of breath. On examination she had systolic murmur in left lower sternal border with respiratory variation. TTE showed mildly dilated right ventricular size. TEE showed left pulmonary vein anomalous connection (Figs. 45.1 and 45.2; Movie 45.1).

45.1  Echo Pearls [1–3] • Isolated partial anomalous pulmonary vein is rare and as part of comprehensive TEE exam, all four pulmonary veins should be searched. • To find right side pulmonary veins, at approximately 55–60 degrees mid-esphageal view, turn probe clockwise and retract slightly. • To identify left side pulmonary veins, at approximately 110 degree, just beside left atrium appendage, turn probe counterclockwise and retract slightly. Electronic Supplementary Material The online version of this chapter (https://doi.org/10.1007/978-3-030-16618-2_45) contains supplementary material, which is available to authorized users.

© Springer Nature Switzerland AG 2019 A. Rohani, Clinical Cases in the Echocardiography Lab, Clinical Cases in Cardiology, https://doi.org/10.1007/978-3-030-16618-2_45

185

186

Chapter 45.  Left Anomalous Pulmonary Vein Connection

Figure 45.1 Left lower pulmonary vein anomalous connection, TEE 121 degree, and mid-esophageal view

Figure 45.2  Abnormal pulmonary vein flow below the baseline

References

187

References 1. Kawasaki H, Oshiro Y, Taira N, et al. Partial anomalous pulmonary venous connection coexisting with lung cancer: a case report and review of relevant cases from the literature. Ann Thorac Cardiovasc Surg. 2016;23(1):31–5. 2. Basalus MW, Said SA, Stassen CM, Fast JH. Clinical and diagnostic features of partially anomalous pulmonary venous connection in an adult female patient: a case report and review of the literature. Neth Heart J. 2011;19(5):256–8. 3. Nath R, Murphy W, Aronson B. Rare case of left upper lobe partial anomalous pulmonary venous connection. J Radiol Case Rep. 2013;7(6):9–14.

Chapter 46 Tetralogy of Fallot

A 25-year-old woman referred to our clinic because of large VSD. Blood pressure was 100/60 mm Hg. Harsh systolic murmur grade 4/6 was heard on apex and left lower left sternal border. TTE revealed a large misaligned ventricular septal defect (VSD) below the tricuspid valve into the outlet septum, with about 50% ascending aorta overriding the inter-ventricular septum. There was an infundibular obstruction with an RV outflow tract systolic peak gradient of 67  mmHg and RV hypertrophy (Figs. 46.1 and 46.2; Movie 46.1).

46.1  Echo Pearls [1, 2] • TOF consists of pulmonary stenosis (PS), Ventricular septal defect (VSD), Right ventricular hypertrophy (RVH) and Overriding Aorta. Electronic Supplementary Material The online version of this chapter (https://doi.org/10.1007/978-3-030-16618-2_46) contains supplementary material, which is available to authorized users.

© Springer Nature Switzerland AG 2019 A. Rohani, Clinical Cases in the Echocardiography Lab, Clinical Cases in Cardiology, https://doi.org/10.1007/978-3-030-16618-2_46

189

190

Chapter 46.  Tetralogy of Fallot

Figure 46.1  TTE PSLX shows overriding of aorta

Figure 46.2  Severe PS, CW Doppler across pulmonary valve

References

191

• Variable degree and location of pulmonary stenosis present in TOF; like as infundibular (subvalvar) dynamic stenosis, supra valvular stenosis, hypoplasia and dysplasia or absent pulmonary valve. • Other abnormalitie such as right aortic arch, rotation of the origin of the coronary arteries, coronary artery anomalies and atrioventricular canal defect are not uncommon.

References 1. Craatz S, Kunzel E, Spanel-Borowski K. Right-sided aortic arch and tetralogy of Fallot in humans-a morphological study of 10 cases. Cardiovasc Pathol. 2003;12:226–32. 2. Need LR, Powell AJ, del Nido P, Geeva T. Coronary echocardiography in tetralogy of Fallot: diagnostic accuracy, resource utilization and surgical implications over 13 years. J Am Coll Cardiol. 2003;36:1371–7.

Chapter 47 Single Ventricle with D-TGA and Pulmonary Stenosis (PS)

A 20-year-old female presented with shortness of breath. On examination she had clubbed fingers. Systemic blood pressure was 110/60  mmHg. Heart has normal S1, S2 with an accentuated pulmonary component of the second heart sound. Chest was clear with no adventitious sound. Electrocardiogram has shown a normal sinus rhythm with left ventricular hypertrophy and right bundle branch block. Transthoracic echocardiography finding include situs solitus of viscera and atria; two separate atrio-ventricular valves opening into a dominant ventricle which is morphologically left ventricle. There was a transposition of great arteries with severe pulmonary valve and supra-valve stenosis (Figs. 47.1 and 47.2; Movie 47.1).

Electronic Supplementary Material The online version of this chapter (https://doi.org/10.1007/978-3-030-16618-2_47) contains supplementary material, which is available to authorized users.

© Springer Nature Switzerland AG 2019 A. Rohani, Clinical Cases in the Echocardiography Lab, Clinical Cases in Cardiology, https://doi.org/10.1007/978-3-030-16618-2_47

193

194

Chapter 47.  Single Ventricle with D-TGA...

Figure 47.1  PSLX, TTE shows parallel great vessel with pulmonary valve malformation

Figure 47.2 Red arrow shows rudimentary chamber, blue arrows show two separate valve valves opening into a dominant left ventricle

47.1  Echo Pearl

195

47.1  Echo Pearl [1–3] 1. Like as every congenital echo focus on segmental approach: • How is abdominal and atrial situs? • How is apical position? • AV relationship, and ventricular-arterial alignment (Tables 47.1 and 47.2) • Aorta or pulmonary vascular bed obstruction (Table 47.3) • Atrio-Ventricular (AV) valve function • Ventricular function • Presence of any shunt Table 47.1  Summarizes ventricular characteristic of univentricular echo report Functionally univentricular heart Ventricular imbalance

Double inlet left ventricle: dominant left ventricle --hypoplastic right ventricle

Double inlet right ventricle: dominant right ventricle, hypoplastic left ventricle

Indeterminate The morphology of type ventricle is not clear.

Table 47.2  Summarizes atrio-ventricular connection subtypes AV connection Double Common inlet Single Morphology may not be adequate to differentiate mitral from tricuspid configuration. May be imperforate, stenotic or regurgitant.

More than 75% of a common AV valve empties into one ventricular chamber.

Mitral atresia or tricuspid valve atresia.

196

Chapter 47.  Single Ventricle with D-TGA...

Table 47.3 Summarizes pulmonary blood flow in univentricular heart No anatomic restriction to Intractable heart failure pulmonary blood flow: without surgery Obstruction to pulmonary blood flow:

Without surgery could survive into adulthood.

• Presence or absence of coarctation of aorta. • Visualization of all surgical shunts (Blalock–Taussig, Waterston, etc.). 2. Univentricular heart defined as both atria relate entirely or almost entirely to one single ventricular chamber.

References 1. Khairy P, Poirier N, Mercier L. Univentricular heart. Circulation. 2007;115(6):800–12. 2. Al-Ethawi AESD, Al-Kaaby BA.  The univentricular heart: revisited. Cir Cardiovasc. 2018;25(3):141–7. 3. Frescura C, Thiene G. The new concept of univentricular heart. Front Pediatr. 2014;2:62.

Chapter 48 Inlet Ventricular Septal Defect (VSD)

A 56-year-old patient presented with shortness of breath and orthopnea. Patient had a history of unrepaired VSD. On examination there was grade IV/VI holosystolic murmur audible in the left parasternal border accompanied by a palpable thrill. Crackles heard in bilateral lower lung field. Echocardiography revealed large size inlet type VSD (high-velocity; maximum systolic velocity 3.9 m/s; pressure gradient of 60 mmHg) (Figs. 48.1 and 48.2; Movie 48.1).

48.1  Echo Pearls [1, 2] Echo report should include following information: 1. Size (tunnel-type VSD is defined as length of the defect divided by width of the defect more than 1.2)

Electronic Supplementary Material The online version of this chapter (https://doi.org/10.1007/978-3-030-16618-2_48) contains supplementary material, which is available to authorized users.

© Springer Nature Switzerland AG 2019 A. Rohani, Clinical Cases in the Echocardiography Lab, Clinical Cases in Cardiology, https://doi.org/10.1007/978-3-030-16618-2_48

197

198

Chapter 48.  Inlet Ventricular Septal Defect (VSD)

Figure 48.1  Inlet VSD (red arrow)

Figure 48.2  TEE long axis view shows VSD

2. Shunt severity: • Volume overload of the left atrium, left ventricle (enlargement of the left heart, is indicative of significant left to right shunt and needs repair).

48.1  Echo Pearls

199

RVOT PA

TV

RA

PA LA

Figure 48.3  Schematic TTE in short axis of great vessel, green area: perimembranous, orange: outlet: infracristal, red: sub pulmonic (outlet: supracristal)

• A continuous wave Doppler velocity of ≥5  m/s across the VSD means small restrictive defect with small left to right shunt. 3. Type: (Fig. 48.3) According to the Society for Thoracic Surgery and the European Association for Cardiothoracic VSD classified into four types: • Type 1 defects involve the infundibular septum, (supracristal, subarterial, subpulmonary, conal, or doubly committed juxta-arterial VSD) with fibrous continuity between the leaflets of the aortic and pulmonary valve. Loss of support of the right cusp causes sagging, herniation or cusp prolapse into the VSD, causing aortic regurgitation.

200

Chapter 48.  Inlet Ventricular Septal Defect (VSD)

• Type 2 defect involve the membranous septum, (cono-ventricular). • Type 3 defect involve the inlet septum, (atrio-­ventricular canal type). • Type 4 defect involve the muscular septum. Atrio-ventricular VSD or Gerbode defect is a left ventricular to right atrial shunt which could be congential or acquired secondary to endocarditis and best visualized on apical four-chamber view; Complications include sinus node dysfunction and tricuspid regurgitation.

References 1. Gabriels C, Backer JD, Pasquet A, Paelinck BP, Morissens M, Helsen F, et  al. Long-term outcome of patients with perimembranous ventricular septal defect: results from the Belgian registry on adult congenital heart disease. Cardiology. 2016;136(3):147–55. 2. Baumgartner H, Bonhoeffer P, De Groot N, Haan F, Deanfield J, Galie N, et al. ESC guidelines for the management of grown-up congenital heart disease. Eur Heart J. 2010;31(23):2915–57.

Chapter 49 Patent Foramen Ovale (PFO)

A 48-year-old female was admitted to our hospital because of right side hemiplegia, and aphasia. On admission she had blood pressure of 105/75  mmHg. Heart and lung examination were unremarkable. Echocardiography imaging and bubble study revealed large widely open PFO (Figs.  49.1 and 49.2). Patient underwent uneventful closure of PFO.

49.1  Echo Pearls [1–3] In an echo report for PFO following information need to be addressed: • • • •

Size of right and left atrial opening. Length of the PFO tunnel. Presence or absence of atrial septum aneurysm. Eustachian valve (or Chiari network) size (distance from the border of IVC).

© Springer Nature Switzerland AG 2019 A. Rohani, Clinical Cases in the Echocardiography Lab, Clinical Cases in Cardiology, https://doi.org/10.1007/978-3-030-16618-2_49

201

202

Chapter 49.  Patent Foramen Ovale (PFO)

Figure 49.1  Multiple bubbles passed through widely open PFO

Figure 49.2  PFO tunnel size and opening size

References

203

References 1. Rohani A. The diagnosis and treatment of paradoxical embolism: a systematic review. Rev Clin Med. 2016;3(4):154–7. 2. Landzberg MJ, Khairy P.  Indications for the closure of patent foramen ovale. Heart. 2004;90(2):219–24. 3. Mattle HP, Meier B, Nedeltchev K. Prevention of stroke in patients with patent foramen ovale. Int J Stroke. 2010;5(2):92–102.

Part VII

Right Ventricle (RV)

Chapter 50 Arrhythmogenic Right Ventricular Cardiomyopathy (ARVC)

Patient is an 18 -year -old young man presents with palpitations and one episode of syncope. Physical examination was normal. ECG shows T wave inversions in anterior leads. Holter monitoring shows a run of non-sustained Ventricular Tachycardia (VT) with left bundle branch morphology. TTE shows dilated RV size and RV apical aneurysm (Fig. 50.1; Movies 50.1 and 50.2).

50.1  Echo Pearls [1–4] ARVC need to be diagnosed based on Task Force criteria and not on structural abnormality only. (Adapted from Marcus et al. [1])

Electronic Supplementary Material The online version of this chapter (https://doi.org/10.1007/978-3-030-16618-2_50) contains supplementary material, which is available to authorized users.

© Springer Nature Switzerland AG 2019 A. Rohani, Clinical Cases in the Echocardiography Lab, Clinical Cases in Cardiology, https://doi.org/10.1007/978-3-030-16618-2_50

207

208

Chapter 50.  Arrhythmogenic Right Ventricular...

Figure 50.1  RV aneurysm (red arrow) TTE apical 4 chambers

50.2  M  ajor Echocardiographic Criteria for ARVC (Arrythmogenic RV Cardiomyopathy) • Severe dilatation and reduction of RV ejection fraction with no (or only mild) LV impairment. • Localized RV aneurysms (akinetic or dyskinetic area with diastolic bulging) • Severe segmental dilatation of the RV and one of the following: 1. PLAX RVOT = 32 mm (corrected for body size [PLAX/ BSA] = 19 mm/m2) 2. PSAX RVOT = 36 mm (corrected for body size [PLAX/ BSA] = 21 mm/m2) 3. Fractional Area Change ≤33%

50.3  M  inor Echocardiographic Criteria for ARVC Regional RV Akinesia or Dyskinesia and one of the following

References

209

1. PLAX RVOT  =  29 to